Health Assessment Practice Questions

Réussis tes devoirs et examens dès maintenant avec Quizwiz!

17. While performing the initial assessment of a client, the client tells the nurse that this is his first hospitalization and that he has no previous surgeries. The nurse should document which of the following? A) Client denies prior hospitalizations and surgeries B) Client has not been hospitalized before nor has he had any surgery C) Client answered no to previous hospitalizations or surgery D) Negative for past hospitalizations

A

24. The nurse has completed the initial assessment of a client and is now performing data analysis. The nurse obtained a blood pressure reading of 114/70 mm Hg. What is this client's pulse pressure? A) 44 mm Hg B) 92 mm Hg C) 114 mm Hg D) 184 mm Hg

A

4. A nurse states, ìHispanic people have no clue about primary prevention of illness.î The nurse is demonstrating which of the following? A) Stereotyping B) Ethnicity C) Cultural incompetence D) Prejudice

A

4. The emergency department has collected extensive data from a client who has presented with a new onset of severe abdominal pain. What nursing action should the nurse perform before proceeding with data analysis? A) Validate the collected data. B) Formulate a nursing diagnosis. C) Make inferences about the data. D) Identify the client's strengths.

A

7. After teaching a group of students about the phases of the nursing process, the instructor determines that the teaching was successful when the students identify which phase as being foundational to all other phases? A) Assessment B) Planning C) Implementation D) Evaluation

A

A newly admitted patient is in acute pain, has not been sleeping well lately, and is having difficulty breathing. How should the nurse prioritize these problems? a. Breathing, pain, and sleep b. Breathing, sleep, and pain c. Sleep, breathing, and pain d. Sleep, pain, and breathing

A

After completing an initial assessment of a patient, the nurse has charted that his respirations are eupneic and his pulse is 58 beats per minute. These types of data would be: A. objective B. reflective C. subjective D. Introspective

A

1. A nurse is preparing to assess a client who is new to the clinic. When beginning the collection of the client database, which of the following actions should the nurse prioritize? A) Establishing a trusting relationship B) Determining the client's strengths C) Identifying potential health problems D) Making clinical inferences

A

10. The nurse is auscultating a client's blood pressure and identifies the portion of the blood pressure cycle reflecting the break in sounds occurring between the first and second sounds. This is known as which of the following? A) Auscultatory gap B) Korotkoff sounds C) Phase V D) Diastolic value

A

11. A nurse is providing in-service training to a group of nurses in a facility that has just begun to use an integrated cued checklist for documentation. Which of the following would the nurse identify as a major advantage of this type of documentation? A) It helps nurses to cluster assessment data. B) It provides lines for the nurses' comments. C) It includes specialized data particular to each client. D) It standardizes data collection.

A

11. An emergency department nurse has utilized the Confusion Assessment Method (CAM) in the assessment of a 79-year-old client with a new onset of urinary incontinence. This assessment tool will allow the nurse to confirm the presence of what health problem? A) Delirium B) Vascular dementia C) Schizophrenia D) Psychosis

A

14. An instructor is reviewing the evolution of the nurse's role in health assessment. The instructor determines that the teaching was successful when the students identify which of the following as the major method used by nurses early in the history of the profession? A) Natural senses B) Biomedical knowledge C) Simple technology D) Critical pathways

A

15. A nurse who provides care on a medical unit utilizes the Alcohol Use Disorders Identification Test (AUDIT) as part of the standard admission protocol. After obtaining a score of 9 from a recently admitted client, the nurse should recognize the possibility of which of the following? A) Hazardous and harmful alcohol use B) Imminent liver disease C) Acute pancreatitis D) Alcoholism

A

17. In the course of performing a client's physical assessment, the nurse has changed from using the diaphragm of the stethoscope to using the bell. The nurse is most likely assessing which of the following? A) Heart sounds B) Bowel sounds C) Breath sounds D) Femoral pulses

A

16. A home care nurse is assessing an older adult's functional status. The nurse should identify which of the following as an instrumental activity of daily living? A) Bathing B) Cooking C) Toileting D) Eating

B

18. A nurse on a busy acute medical unit asks a clinical educator for suggestions on how to best develop expertise in using diagnostic reasoning skills to arrive at correct conclusions. Which of the following statements would be most appropriate? A) "You need to cluster the data more rapidly." B) "This skill comes with accumulating experience." C) "Try to be more efficient in documenting the data." D) "This is a skill that only comes with an advanced practice designation."

B

19. A nurse is preparing to perform the physical examination of an adult client who has presented to the clinic for the first time. Which of the following statements should guide the nurse's use of a stethoscope during this phase of assessment? A) Auscultation can be performed through clothing. B) The diaphragm should be held firmly against the body part. C) The bell of the stethoscope can best detect bowel sounds. D) Use of the bell is reserved for advanced practice nurses.

B

19. A nurse is working on an acute neurological unit. Which assessment form would the nurse most likely use to document assessment data? A) Open-ended form B) Focused assessment form C) Frequent assessment form D) Ongoing assessment form

B

19. The nurse is assessing the client's perception of pain and the client's description of its intensity and quality. Which dimension of pain is the nurse evaluating? A) Physical B) Sensory C) Behavioral D) Cognitive

B

2. A client admits to the nurse that she feels guilty for not providing more direct care for her ill mother. According to Freud, the moral component of this client's feelings results from which of the following? A) Defense mechanisms B) The superego C) The id D) The ego

B

2. A client has presented to the emergency department (ED) with complaints of abdominal pain. Which member of the care team would most likely be responsible for collecting the subjective data on the client during the initial comprehensive assessment? A) Gastroenterologist B) ED nurse C) Admissions clerk D) Diagnostic technician

B

2. A nurse is interpreting and validating information from an older adult client who has been experiencing a functional decline. The nurse is in which phase of the interview? A) Introductory B) Working C) Summary D) Closing

B

20. When attempting to assess a client's pain, which of the following actions should the nurse perform first? A) Observe behaviors in the client. B) Obtain a client self-report. C) Search for possible causes of pain. D) Ask family members about the client's pain.

B

24. A nurse is using the Katz Activities of Daily Living tool to assess an older adult's functional status. What question will the nurse include in this assessment? A) ìWho generally prepares your meals and snacks?î B) ìDo you require any assistance when showering or bathing?î C) ìDo you feel like you have enough support from your family?î D) ìAre you able to shop for your own groceries?î

B

24. The nurse is performing a health assessment on a community-dwelling client who is recovering from hip replacement surgery. Which of the following actions should the nurse prioritize during assessment? A) Focus the assessment on the client as a member of her age group. B) Interpret the information about the client in context. C) Corroborate the client's statements with trusted sources. D) Gather information from a variety of sources.

B

24. The nurse's assessment suggests that a 10-year-old has failed to achieve Erikson's central task of this stage of development. What nursing diagnosis should most likely be included in the child's plan of care? A) Risk for injury B) Chronic low self-esteem C) Fear D) Disturbed thought processes

B

27. The nurse is percussing the area over the client's lungs and hears a loud, low-pitched, hollow sound. The nurse documents this finding as which of the following? A) Flatness B) Resonance C) Tympany D) Dullness

B

4. A nurse is interviewing a 22-year-old client of the campus medical clinic. Which nonverbal behavior should the nurse adopt to best facilitate communication during this phase of assessment? A) Standing while the client is seated B) Using a moderate amount of eye contact C) Sitting across the room from the client D) Minimizing facial expressions

B

6. A client has presented to the emergency department and is having difficulty describing her vague sensation of physical discomfort and unease. How can the nurse best elicit meaningful assessment data about the nature of the client's complaint? A) Ignore the complaint for now and return to it later in the assessment. B) Provide a laundry list of descriptive words. C) Restate the question using simpler terms. D) Wait in silence until the client can determine the correct words.

B

8. The nurse begins the physical examination of a newly admitted client by assessing the client's mental status. What is the nurse's best rationale for performing the mental status exam early in the assessment? A) The client will be less anxious early, providing the nurse with more accurate and reliable data. B) The exam can provide clues about the validity of the client's responses now and throughout. C) The exam provides data about mental health problems that the client may be afraid to report. D) The client's fears about having a serious illness may be alleviated by the results of the exam.

B

9. The nurse is working in an ambulatory care clinic that is located in a busy, inner-city neighborhood. Which client would the nurse determine to be in most need of an emergency assessment? A) A 14-year-old girl who is crying because she thinks she is pregnant B) A 45-year-old man with chest pain and diaphoresis for 1 hour C) A 3-year-old child with fever, rash, and sore throat D) A 20-year-old man with a 3-inch shallow laceration on his leg

B

9. Which of the following would be most important for the nurse to do when assessing a client's blood pressure? A) Palpate the pulsations of the ulnar artery. B) Hold the client's arm slightly flexed with palm down. C) Inflate the cuff 30 mm Hg above where the radial pulse disappears. D) Deflate the cuff about 5 mm Hg per second.

C

A patient tells the nurse that he is very nervous, is nauseated, and feels hot. These types of data would be: A. Objective B. Reflective C. subjective D. Introspective

C

14. A medical nurse has completed the review of systems component of the client's health history. Which assessment finding should the nurse document under the review of systems? A) "High school diploma plus 2 years of college" B) "Caregiver reliable source of information" C) "Menarche at age 13" D) "Lungs clear to auscultation bilaterally"

C

3. A 71-year-old woman has been admitted to the hospital for a vaginal hysterectomy, and the nurse is collecting subjective data prior to surgery. Which statement by the nurse could be construed as judgmental? A) "How often do your adult children typically visit you?" B) "Your husband's death must have been very difficult for you." C) "You must quit smoking because it affects others, not only you." D) "How would you describe your feelings about getting older?"

C

3. A client who had a mastectomy is being discharged home on postoperative day 1. Knowing that the client lives alone, which data would be most important for the nurse to validate for this client? A) If the client has transportation for follow-up appointments B) If the client usually functions independently C) What support systems are in place to assist the client D) If the client has a religious belief regarding illness

C

5. During an assessment of an elderly client, the nurse notes a decrease in pupil size and a slowed reaction of the pupil to light. Accommodation and convergence are normal. Based on these findings, which of the following should the nurse emphasize with client education? A) Use drops to prevent dryness B) Wear sunglasses outdoors C) Avoid driving at night D) Obtain an eye examination

C

8. A 54-year-old client is receiving a follow-up assessment in a clinic, following abnormal findings on her recent mammogram. Which of the following statements best reflects appropriate documentation by the nurse? A) ìClient depressed because of fear of breast biopsyî B) ìClient with lower back painî C) ìClient has unkempt appearance and avoids eye contactî D) ìClient has good lung sounds in right and left lungsî

C

11. The nurse is assessing a client whose chronic pain is poorly controlled. Which assessment finding should the nurse expect under these circumstances? A) Decreased heart rate B) Hypoglycemia C) Increased urinary output D) Decreased gastric motility

D

13. A nurse is comparing the subjective data and objective data obtained from an assessment of a client who is thought to have hepatitis A. This nurse's comparison will achieve what benefit to this client's care? A) Formulation of nursing diagnoses B) Identification of missing data C) Determination of documentation form to use D) Validation of data

D

22. The nurse assists a client into the dorsal recumbent position. Assessment of which area is contraindicated when the client is in this position? A) Chest B) Head C) Peripheral pulses D) Abdomen

D

28. A nurse is providing care to a client who has been in a motor vehicle accident and who has facial lacerations and a pelvic fracture. How can the nurse best determine the reliability and accuracy of data obtained during a pain assessment? A) Ask the primary care provider to validate the assessment data. B) Compare the findings to the client's preinjury level of health. C) Compare the findings to the most recent previous pain assessment. D) Validate the assessment data with the client.

D

8. The following information is recorded in the health history: "Patient denies chest pain, palpitations, orthopnea, and paroxysmal nocturnal dyspnea." Which category does it belong to? A) Chief complaint B) Present illness C) Personal and social history D) Review of systems

D; Ans: D Chapter: 01 Page and Header: 6, The Comprehensive Adult Health History Feedback: Review of systems documents the presence or absence of common symptoms related to each major body system. The absence of cardiac symptoms is listed in the above example.

A 16-year-old boy has just been admitted to the unit for overnight observation after being in an automobile accident. What is the nurses best approach to communicating with him? a. Use periods of silence to communicate respect for him. b. Be totally honest with him, even if the information is unpleasant. c. Tell him that everything that is discussed will be kept totally confidential. d. Use slang language when possible to help him open up.A 16-year-old boy has just been admitted to the unit for overnight observation after being in an automobile accident. What is the nurses best approach to communicating with him?

b

When observing a patients verbal and nonverbal communication, the nurse notices a discrepancy. Which statement is true regarding this situation? The nurse should: a. Ask someone who knows the patient well to help interpret this discrepancy. b. Focus on the patients verbal message, and try to ignore the nonverbal behaviors. c. Try to integrate the verbal and nonverbal messages and then interpret them as an average. d. Focus on the patients nonverbal behaviors, because these are often more reflective of a patients true feelings.

d

In the ________________ model_, the focus of the health professional is on helping the consumer choose a healthier lifestyle.

health promotion

The nurse has implemented several planned interventions to address the nursing diagnosis of acute pain. Which would be the next appropriate action? a. Establish priorities. b. Identify expected outcomes. c. Evaluate the individuals condition, and compare actual outcomes with expected outcomes. d. Interpret data, and then identify clusters of cues and make inferences.

c

16. The nurse is observing a client for evidence of pain. Which of the following would most likely lead the nurse to suspect that the client may be experiencing pain? A) Frequent questioning B) Slumped posture C) Eye contact D) Periodic position changes

B

13. A nurse is applying the diagnostic reasoning process in the care of a client. What is the correct sequence of the steps that the nurse should perform? A) Check for defining characteristics. B) Draw inferences. C) Propose possible nursing diagnoses. D) Identify abnormal data and strengths. E) Cluster data.

D, E, B, C, A

6. The nurse is preparing to assess an older adult client's near vision. Which of the following pieces of equipment would be most appropriate for the nurse to use? A) Newspaper B) Snellen chart C) Ophthalmoscope D) Penlight

A

11. A male Hispanic client describes the fact that he mixed hot and cold foods, causing them to lump together and ìget stuck in his intestines,î causing diarrhea and abdominal pain. The nurse would document this as which of the following? A) Empacho B) Susto C) Mal ojo D) Mal puesto

A

12. A group of nursing students is reviewing the purposes of assessment documentation in preparation for a class discussion. The students demonstrate understanding of the information when they identify which of the following as one of the primary purposes? A) It provides a chronologic source of client assessment data. B) It creates a database for care that was not rendered to the client. C) It replaces the client acuity classification system. D) It directly formulates the nursing diagnoses.

A

12. A nurse observes the posture of a male client and finds him leaning forward and bracing himself while sitting on the exam table. Which of the following would the nurse most likely suspect? A) Chronic obstructive pulmonary disease B) Neurological deficit C) Metabolic disorder D) Vestibular disorder

A

13. A client has presented for care with complaints of persistent lower back pain. When using the mnemonic COLDSPA, which question should the nurse use to evaluate the "P"? A) "What makes it worse?" B) "When did it start?" C) "How does it feel?" D) "How would you rate your pain?"

A

13. The nurse is assessing a client's pain. Which question would be most appropriate to ask the client when the goal is to identify precipitating factors that might have exacerbated the pain? A) ìWhat were you doing when the pain first stated?î B) ìDo concurrent symptoms accompany the pain?î C) ìWhen did the pain start?î D) ìIs the pain continuous or intermittent?î

A

14. After teaching a group of students about geriatric syndromes, the instructor determines that the teaching was successful when the students identify which of the following as an example? A) Confusion B) Pneumonia C) Heart failure D) Renal failure

A

15. When assessing pain in an older adult client who is alert and oriented, which assessment tool would be most appropriate to use? A) Numerical rating scale B) Faces Pain Scale-Revised C) FLACC Scale D) Graphic rating scale

A

16. A group of nurses are reviewing information about the potential opportunities for nurses who have advanced assessment skills. When discussing phenomena that have contributed to these increased opportunities, what should the nurses identify? A) Expansion of health care networks B) Decrease in client participation in care C) The shrinking cost of medical care D) Public mistrust of physicians

A

7. A nurse has completed a client's initial assessment and is now interpreting and making inferences from the data. The nurse is involved in which phase of the nursing process? A) Analysis B) Planning C) Implementation D) Evaluation

A

17. A nurse is interviewing an elderly client and begins the interview by evaluating the client's mental status. The nurse does this based on an understanding of which of the following? A) The aging brain is more easily affected by pathology. B) Older clients have decreased intellectual capacity. C) The brain is the last organ to experience an age-related decline. D) The client is always the most reliable person to provide the data.

A

18. A nurse is assessing an Asian client and observes several reddened and bruised areas on the skin. Further assessment reveals that the client was using cupping to treat back pain. The nurse understands this as which of the following? A) Placing heated glass jars on the skin that are allowed to cool B) Rubbing ointment into the skin with a spoon C) Attaching smoldering herbs to acupuncture needles D) Placing warm burning herbs directly on the skin

A

18. What assessment finding would most clearly suggest to the nurse that a young adult client has failed to attain normal development within Piaget's framework? A) The client has difficulty understanding abstract reasoning in written form. B) The client has a recent history of tumultuous interpersonal relationships. C) The client is often defiant toward authority figures. D) The client is unwilling to accept responsibilities in the workplace.

A

19. A community health nurse is assessing an older adult client in the client's home. When the nurse is gathering subjective data, which of the following would the nurse identify? A) The client's feelings of happiness B) The client's posture C) The client's affect D) The client's behavior

A

19. A nurse is creating a genogram of a client's family health history. The nurse should use which of the following symbols to denote the client's female relatives? A) Circle B) Square C) Triangle D) Rectangle

A

2. A nurse is admitting a client who is from another culture. Prior to caring for a client from another culture, the nurse should place primary importance on which action? A) Examining personal biases and prejudices B) Researching characteristics of the specific culture C) Asking colleagues about ways to approach the client D) Developing awareness of the culture's health practices

A

2. A nurse is assessing a female client whose worsening sciatica has prompted her to seek care. Which of the client's following statements would the nurse most likely need to validate? A) ìI don't generally have problems with pain.î B) ìI feel very weak and tired right now.î C) ìI've had two cesarean deliveries.î D) ìMy mother died of breast cancer in her sixties.î

A

2. A nurse is preparing to assess an adult client's body temperature. At which time of the day would the nurse expect to obtain the lowest body temperature? A) Early morning B) Early afternoon C) Late afternoon D) Late evening

A

2. A young adult client has come to the clinic for her scheduled Pap (Papanicolaou) test and pelvic examination. The nurse is implementing actions to help reduce a client's anxiety during the physical exam. Which of the following would be most appropriate? A) Ensuring client's privacy by providing an examination gown B) Providing a comfortable, warm room temperature C) Arranging exam equipment on a bedside tray table D) Explaining why standard precautions are being used

A

20. The nurse has assessed the thorax and lungs of an elderly client, as well as reviewing the results of lung function testing. Which of the following findings should the nurse attribute to possible pathology rather than expected, age-related changes? A) Respiratory rate of 30 breaths per minute B) Decreased vital capacity C) Increased residual volume D) Presence of a slight barrel chest

A

21. A nurse's reflection of his practice reveals that he tends to see his own culture as the ìgold standardî to which all other cultures should aspire. This nurse should create learning goals to address what phenomenon? A) Ethnocentrism B) Unconscious incompetence C) Stereotyping D) Acculturation

A

22. A surgical client's pain has become increasingly severe overnight, and she has received her maximum current doses of analgesics. The nurse has consequently phoned the surgeon to obtain a new order for analgesia. After the surgeon tells the nurse the new order, how should the nurse best validate this information? A) Read the order back to the surgeon for confirmation. B) Compare the order with the standard timing and dosage of the analgesic. C) Compare the order to the client's existing medication administration record (MAR). D) Have another nurse read the order that the nurse has transcribed.

A

22. An emergency department nurse is assessing a client's complaint of upper abdominal pain. Using the COLDSPA mnemonic, with what assessment question would the nurse begin? A) ìCan you describe to me how your pain feels?î B) ìHow would you rate your pain on a 10-point scale?î C) ìIs your pain affecting your ability to cope?î D) ìWould you describe your pain as acute, or as chronic?î

A

23. A nurse has admitted a client to the medical unit who has just been diagnosed with endocarditis secondary to IV drug use. The nurse has completed the collection of objective and subjective data. What question should guide the next step in the nurse's data analysis? A) "What are this client's strengths?" B) "What is this client's prognosis?" C) "Why does this client use opioids?" D) "What are this client's hopes for the future?"

A

23. A nurse is completing an assessment that will involve gathering subjective and objective data. Which of the following assessment techniques will best allow the nurse to collect objective data? A) Inspection B) Therapeutic communication C) Interviewing D) Active listening

A

23. A patient is at the clinic to have her blood pressure checked. She has been coming to the clinic weekly since she changed medications 2 months ago. The nurse should: a. Collect a follow-up data base and then check her blood pressure. b. Ask her to read her health record and indicate any changes since her last visit. c. Check only her blood pressure because her complete health history was documented 2 months ago. d. Obtain a complete health history before checking her blood pressure because much of her history information may have changed.

A

25. The nurse is assessing an older adult client's vaccination history. This aspect of the client's history will have a significant bearing on her risk for what health problem? A) Pneumonia B) Urinary tract infections C) Cellulitis D) Tuberculosis

A

25. What statement by a middle-aged adult would most clearly suggest successful achievement of Erikson's central task during this stage of development? A) ìI'm doing a lot of volunteering in order to give back to the community.î B) ìI've started to exercise more regularly so that I don't put on extra weight.î C) ìI socialize with my coworkers a lot more than I did when I was younger.î D) ìOverall, my marriage is likely stronger than it was when we first got married.î

A

26. A 21-year-old woman has been admitted to the emergency department following an accident that is suspected of being a suicide attempt. When assessing the client's perceptions, what question should the nurse ask the client? A) ìHow would you describe your health these days?î B) ìAre you able to smell and taste as well as you've been able to in the past?î C) ìIf you found a stamped envelope on the street, what would you do?î D) ìCan you tell me the circumstances surrounding your accident?î

A

26. A female client with advanced-stage vascular dementia has been showing signs of pain over the past several hours. The nurse is unable to obtain a self-report from the client due to her cognitive impairment. When applying the Hierarchy of Pain Assessment Techniques, how should the nurse proceed with assessment? A) Search for potential causes of pain. B) Ask the client's family if they believe she is in pain. C) Perform interventions as if the client were in pain. D) Use a visual assessment tool rather than a verbal tool.

A

26. A nurse has assessed a client who was admitted to the medical unit to treat acute complications of type 1 diabetes. During the assessment, the client admitted that his blood sugar monitoring when he is at home is ìa bit sporadic.î How should the nurse best respond to this assessment finding? A) Identify a nursing diagnosis of Ineffective Health Maintenance. B) Identify a collaborative problem that should involve the occupational therapist. C) Make a referral to the unit's social work department. D) Reassess the client's blood glucose level.

A

26. A nurse is providing care for a client who has longstanding type 2 diabetes. In recent days, the client's blood glucose levels have been higher and more volatile than usual. After drawing this inference, the nurse should take what action? A) Make appropriate referrals B) Assess the client more frequently C) Document the medical diagnosis of hyperglycemia D) Beginning collecting subjective data

A

26. A nurse is reviewing the four basic physical examination techniques and their sequence prior to receiving a new client from postanesthetic recovery. The nurse should plan to perform which technique first? A) Inspection B) Palpation C) Percussion D) Auscultation

A

27. A community health nurse is conducting a home visit to a client who requires wound care. The nurse observes that the client is diaphoretic and wishes to measure the client's temperature. The nurse asks if the client has a thermometer in her home, and she states that she owns an ìear thermometer.î What principle should guide the nurse's use of a tympanic thermometer? A) Tympanic temperature is slightly higher than oral temperature. B) Tympanic temperature is only used if all other methods are unavailable. C) Tympanic temperature varies more widely than oral, rectal, and axillary temperatures. D) In adults, tympanic temperature is equal to axillary temperature.

A

27. A nurse in the emergency department is utilizing the SAD PERSONAS assessment guide during the mental status assessment of a client. What is the most likely rationale for the nurse's choice of this assessment tool? A) The client may have a high risk for suicide. B) The client may have major depression. C) The client may have schizophrenia or psychosis. D) The client may be using alcohol excessively.

A

27. A nurse is assessing an African-American client who has a longstanding diagnosis of hypertension. The nurse should be aware that the client may experience a greater-than- average effect of what medication? A) A diuretic B) An angiotensin-converting enzyme inhibitor C) A calcium channel blocker D) A beta-adrenergic blocker

A

27. An older adult client has received a diagnosis of stress incontinence, and the nurse is planning the client's subsequent care. What health education is most relevant to this client's needs? A) Pelvic floor strength training and activity management B) Appropriate use of incontinence pads and dietary modifications C) Management of fluid and electrolyte intake D) Aseptic technique for intermittent catheterization and fluid restriction

A

27. The nurse is assessing an adult client for the presence of Piaget's formal operations stage of development. What assessment question should the nurse ask the client? A) ìHow do you usually go about making difficult decisions?î B) ìDo you consider yourself to be an intelligent person?î C) ìHow would you describe your relationship with authority figures?î D) ìIn relationships, do you consider yourself to be a 'giver' or a 'taker'?î

A

27. The nurse is utilizing the Health Belief Model in the care of a client whose type 1 diabetes is inadequately controlled. When implementing this model, the nurse should begin by assessing which of the following? A) The client's motivation for change B) The client's medical comorbidities C) The client's learning style D) The client's prognosis for recovery

A

28. A 20-year-old female client has presented to the clinic, and the nurse is preparing to perform a comprehensive assessment. The client states, ìI'd really like to have my mom in the room. That's okay, isn't it?î How should the nurse best respond to the client's request? A) ìOf course. There's a chair in the exam room where she can sit.î B) ìThat's no problem. I'll just have to get you to sign a privacy waiver first.î C) ìThat's fine, but be aware that some of the examinations might be embarrassing for you or her.î D) ìIt's best to undergo the examination alone in order to make sure I get accurate data, but if you really want her present, we can do that.î

A

29. During the nurse's assessment of the client's exercise and activity habits, the client laughs and then states, "Unless you're including channel surfing, I don't really do much of anything." How should the nurse best follow up this client's statement? A) Briefly describe some of the potential benefits of regular exercise. B) Ask the client if he understands the risk factors for heart disease and diabetes. C) Explain to the client that he should be performing aerobic exercise for 20 to 30 minutes at least three times a week. D) Document the nursing diagnosis of Risk for Activity Intolerance related to sedentary lifestyle.

A

29. The nurse palpates a client's pulse and notes that the rate is 71 beats per minute, with an irregular rhythm. How should the nurse follow up this assessment finding? A) Auscultate the client's apical pulse. B) Palpate the client's ulnar pulse. C) Administer a dose of nitroglycerin. D) Reposition the client in a side-lying position.

A

3. The nurse is preparing to assess a client's vital signs. Which vital sign should the nurse assess first? A) Temperature B) Pulse C) Respiration D) Blood pressure

A

3. The nurse utilizes the Depression Questionnaire on a client who has recently moved to a long-term care facility. The total score is 22. Which of the following would be most appropriate for the nurse to do next? A) Refer for further evaluation. B) Evaluate benefits versus risks of a mental health label. C) Assess further for dementia. D) Document this as a normal score.

A

30. A nurse admits to a colleague, ìI sometimes tend to avoid clients from other cultures because it's awkward and it's usually frustrating for me and for the client.î This nurse is likely lacking in what construct of cultural competency? A) Cultural desire B) Cultural knowledge C) Cultural health D) Cultural harmony

A

30. A nurse has collecting extensive data during a client assessment and is performing the first step in the process of data analysis. Successful completion of this step requires the nurse to do which of the following? A) Differentiate between expected findings and abnormal findings. B) Validate nursing diagnoses with the client and the client's family. C) Integrate the client's medical diagnosis with nursing diagnoses. D) Perform health promotion education.

A

30. The nurse is assessing an older adult client's mental status. Consistently, the client pauses after the nurse poses a question, but then the client provides a response that is correct or appropriate. How should the nurse best interpret this characteristic of the client? A) Slight delays in mental processing are normal in older adults. B) The client may be trying to anticipate the nurse's desired response. C) The client is displaying a sign of early Alzheimer's disease. D) The client may be experiencing an early sign of delirium.

A

4. An infant was removed from her home by social services because of the dangerous and neglectful conditions that existed. According to Erikson, failure of the infant to resolve the central crisis of infancy may lead to what personality characteristics later in life? A) Suspicion and fear B) Aggression and antagonism C) Dependency and relational entanglement D) Depression and introversion

A

4. The nurse notes that an older adult client is wearing multiple layers of clothing on a warm fall day. Which of the following would be the nurse's priority assessment at this time? A) Asking whether the client often feels cold B) Assessing the client's developmental level C) Reviewing the client's culture for possible influence D) Observing the client's overall hygiene

A

4. The nurse who provides care at an ambulatory clinic is preparing to meet a client and perform a comprehensive health assessment. Which of the following actions should the nurse perform first? A) Review the client's medical record. B) Obtain basic biographic data. C) Consult clinical resources explaining the client's diagnosis. D) Validate information with the client.

A

5. A nurse has completed a client's initial assessment and is preparing to identify abnormal data and the client's strengths. Successful completion of this phase of the nursing process most requires which of the following? A) Knowledge of anatomy and physiology B) Awareness of the client's medical prognosis C) Inferences about the client D) Knowledge about the referral process

A

5. A nurse has documented the nursing history and physical examination of a client. This health information is best described as which of the following? A) Subjective data and objective data B) Interpretation and inference C) Observation and inspection D) Data and results

A

5. A nurse is assessing a client of East Asian descent. Which biological variation would the nurse expect? A) Dry cerumen in the client's ears B) Profuse perspiration in the client's axillary area C) Strong body odor D) Longer eustachian tubes

A

5. The nurse is applying Piaget's theory of development to a client's health history. This approach to analysis will prioritize what activity on the part of the client? A) Learning B) Imitating C) Indulging D) Desiring

A

6. The nurse is analyzing the data obtained from a client interview. When applying the principles of Kohlberg's theory of development, the nurse should prioritize data related to what domain? A) The client's moral behavior B) The client's relationships C) The client's health D) The client's sexual identity

A

7. A nurse is eliciting a client's health history and the client asks, "Can I take the herb ginkgo biloba with my other medications?" What action would be best if the nurse is unsure of the answer? A) Promise to find out the information for the client. B) Change the subject and return to this topic later. C) Teach the client to only take prescribed medications. D) Encourage the client to ask the pharmacist or primary care provider.

A

7. When auscultating the heart of an elderly client, the nurse detects a soft systolic murmur at the base of the heart. The nurse understands that this is most likely the result of which of the following? A) Calcification of the aortic and mitral valves B) Accumulation of amyloid in the pacemaker cells C) Enlargement of the heart muscle D) Regurgitation through a stenotic valve

A

8. The nurse has completed the comprehensive health assessment of a client who has been admitted for the treatment of community-acquired pneumonia. Following the completion of this assessment, the nurse periodically performs a partial assessment primarily for which reason? A) Reassess previously detected problems B) Provide information for the client's record C) Address areas previously omitted D) Determine the need for crisis intervention

A

8. The nurse is preparing to assess the mental status of a 90-year-old client who is being admitted to the hospital from a long-term care facility. Which of the following should the nurse assess first? A) The client's sensory abilities B) The client's general intelligence C) The presence of any phobias D) The client's judgment and insight

A

9. A female client is told that she needs a pelvic exam and Papanicolaou (Pap) smear. She says ìAbsolutely not! There's no way I'll let you do that to me!î Which response by the nurse would be most appropriate? A) Explain the importance of the pelvic exam and Pap smear, but respect the client's wishes and omit the exam. B) Tell the client that this is the only way she can be checked for cancer. C) Ask the client if she would prefer another practitioner to perform the exam. D) Proceed with the pelvic exam and document the client's protests in the health record.

A

9. A nurse provides care in a rural hospital that serves a community that has few minority residents. When interviewing a client from a minority culture, the nurse has enlisted the assistance of a "culture broker." How can this individual best facilitate the client's care? A) By interpreting the client's language and culture B) By evaluating the client's culturally based health practices C) By teaching the client about health care D) By making the client feel comfortable and safe

A

9. Based on the analysis of assessment data from a client with pain, the nurse writes a health promotion diagnosis. Which of the following diagnoses would be most appropriate? A) Readiness for enhanced spiritual well-being related to coping with prolonged physical pain B) Risk for activity intolerance related to chronic pain and immobility C) Bathing self-care deficit related to severe pain D) Chronic pain related to chronic inflammatory process of rheumatoid arthritis

A

Barriers to incorporating EBP include: a. Nurses lack of research skills in evaluating the quality of research studies. b. Lack of significant research studies. c. Insufficient clinical skills of nurses. d. Inadequate physical assessment skills.

A

Expert nurses learn to attend to a pattern of assessment data and act without consciously labeling it. These responses are referred to as: a.Intuition. b. The nursing process. c. Clinical knowledge. d. Diagnostic reasoning.

A

The nurse knows that developing appropriate nursing interventions for a patient relies on the appropriateness of the __________ diagnosis. a. Nursing b. Medical c. Admission d. Collaborative

A

The patients record, laboratory studies, objective data, and subjective data combine to form the: a.Data base. b. Admitting data. c. Financial statement. d. Discharge summary.

A

1. A patient presents for evaluation of a sharp, aching chest pain which increases with breathing. Which anatomic area would you localize the symptom to? A) Musculoskeletal B) Reproductive C) Urinary D) Endocrine

A; Ans: A Chapter: 02 Page and Header: 27, Assessment and Plan: The Process of Clinical Reasoning Feedback: Chest pain may be due to a musculoskeletal condition, such as costochondritis or intercostal muscle cramp. This would be worsened by motion of the chest wall. Pleuritic chest pain is also a sharp chest pain which increases with a deep breath. This type of pain can occur with inflammation of the pleura from pneumonia or other conditions and pulmonary embolus.

3. Is the following information subjective or objective? Mr. M. has shortness of breath that has persisted for the past 10 days; it is worse with activity and relieved by rest. A) Subjective B) Objective

A; Ans: A Chapter: 01 Page and Header: 6, Differences Between Subjective and Objective Data Feedback: This is information given by the patient about the circumstances of his chief complaint. It does not represent an objective observation by the examiner.

7. The following information is recorded in the health history: "I feel really tired." Which category does it belong to? A) Chief complaint B) Present illness C) Personal and social history D) Review of systems

A; Ans: A Chapter: 01 Page and Header: 6, The Comprehensive Adult Health History Feedback: The chief complaint is an attempt to quote the patient's own words, as long as they are suitable to print. It is brief, like a headline, and further details should be sought in the present illness section. The above information is a chief complaint.

10. The following information is best placed in which category? "The patient had a stent placed in the left anterior descending artery (LAD) in 1999." A) Adult illnesses B) Surgeries C) Obstetrics/gynecology

A; Ans: A Chapter: 01 Page and Header: 9, Past History Feedback: The adult illnesses category is reserved for chronic illnesses, significant hospitalizations, significant injuries, and significant procedures. A stent is a major procedure but does not involve a surgeon.

11. The following information is best placed in which category? "The patient was treated for an asthma exacerbation in the hospital last year; the patient has never been intubated." A) Adult illnesses B) Surgeries C) Obstetrics/gynecology D) Psychiatric

A; Ans: A Chapter: 01 Page and Header: 9, Past History Feedback: This is information about a significant hospitalization and should be placed in the adult illnesses section. If the patient is being seen for an asthma exacerbation, you may consider placing this information in the present illness section, because it relates to the chief complaint at that visit.

18. You have recently returned from a medical missions trip to sub-Saharan Africa, where you learned a great deal about malaria. You decide to use some of the same questions and maneuvers in your "routine" when examining patients in the midwestern United States. You are disappointed to find that despite getting some positive answers and findings, on further workup, none of your patients has malaria except one, who recently emigrated from Ghana. How should you next approach these questions and maneuvers? A) Continue asking these questions in a more selective way. B) Stop asking these questions, because they are low yield. C) Question the validity of the questions. D) Ask these questions of all your patients.

A; Ans: A Chapter: 02 Page and Header: 38, The Challenges of Clinical Data Feedback: The predictive value of a positive finding depends upon the prevalence of a given disease in a population. The prevalence of malaria in the Midwest is almost zero, except in people immigrating from areas of high prevalence. You will waste time and resources applying these questions and maneuvers to all patients. It would be wise to continue applying what you learned to those who are from areas of high prevalence of a given disease. Likewise, physicians from Ghana should not ask about signs or symptoms of multiple sclerosis, as it is found almost exclusively in northern latitudes. You will learn to tailor your examination to the population you are serving.

10. A 62-year-old teacher presents to the clinic for evaluation of the following symptoms: fever, headache, sinus congestion, sore throat, green nasal discharge, and cough. This cluster of symptoms is best explained by: A) One disease process B) More than one disease process

A; Ans: A Chapter: 02 Page and Header: 38, The Challenges of Clinical Data Feedback: This cluster of symptoms is most consistent with sinusitis. The chance that all of these symptoms are caused by multiple synchronous conditions in the same patient is much less than the possibility of having one problem which accounts for all of them.

12. Ms. Washington is a 67-year-old who had a heart attack last month. Now she complains of shortness of breath and not being able to sleep in a flat position (orthopnea). On examination you note increased jugular venous pressure, an S3 gallop, crackles low in the lung fields, and swollen ankles (edema). This is an example of a: A) Pathophysiologic problem B) Psychopathologic problem

A; Ans: A Chapter: 02 Page and Header: 38, The Challenges of Clinical Data Feedback: This is an example of a pathophysiologic problem because Ms. Washington's symptoms are consistent with a pathophysiologic process. The heart attack reduced the ability of her heart to handle her volume status and subsequently produced the many features of congestive heart failure.

17. You are growing fatigued of performing a maneuver on examination because you have never found a positive and are usually pressed for time. How should you next approach this maneuver? A) Use this test when you have a higher suspicion for a certain correlating condition. B) Omit this test from future examinations. C) Continue doing the test, but rely more heavily on laboratory work and diagnostics. D) Continue performing it on all future examinations.

A; Ans: A Chapter: 02 Page and Header: 38, The Challenges of Clinical Data Feedback: This is an example of a specific test that lacks sensitivity. With this scenario, when you finally find a positive, you might be very certain that a given condition is present. We generally develop our examinations to fit our clinical experiences. Sensitive tests are performed routinely on the screening examination, while specific tests are usually saved for the detailed or "branched" examinations. Branched examinations are further maneuvers we can perform to investigate positive findings on our screening examinations. Save this type of maneuver to confirm your hypothesis. All of this information also applies to history questions.

15. When you enter your patient's examination room, his wife is waiting there with him. Which of the following is most appropriate? A) Ask if it's okay to carry out the visit with both people in the room. B) Carry on as you would ordinarily. The permission is implied because his wife is in the room with him. C) Ask his wife to leave the room for reasons of confidentiality. D) First ask his wife what she thinks is going on.

A; Ans: A Chapter: 03 Page and Header: 60, Learning About the Patient: The Sequence of the Interview Feedback: Even in situations involving people very familiar with each other, it is important to respect individual privacy. There is no implicit consent merely because he has allowed his wife to be in the room with him. On the other hand, it is inappropriate to assume that his wife should leave the room. Remember, the patient is the focus of the visit, so it would be appropriate to allow him to control who is in the room with him and inappropriate to address his wife first. Although your duty is to the patient, you may get optimal information by offering to speak to both people confidentially. This situation is analogous to an adolescent's visit.

17. You have just asked a patient how he feels about his emphysema. He becomes silent, folds his arms across his chest and leans back in his chair, and then replies, "It is what it is." How should you respond? A) "You seem bothered by this question." B) "Next, I would like to talk with you about your smoking habit." C) "Okay, let's move on to your other problems." D) "You have adopted a practical attitude toward your problem."

A; Ans: A Chapter: 03 Page and Header: 60, Learning About the Patient: The Sequence of the Interview Feedback: You have astutely noted that the patient's body language changed at the time you asked this question, and despite the patient's response, you suspect there is more beneath the surface. Maybe he is afraid of being browbeaten about his smoking, maybe a relative has recently died from this disorder, or maybe a friend told him 20 years ago that he would eventually get emphysema. Regardless, by sharing your observation and leaving a pause, he may begin to talk about some issues which are very important to him.

4. Jason is a 41-year-old electrician who presents to the clinic for evaluation of shortness of breath. The shortness of breath occurs with exertion and improves with rest. It has been going on for several months and initially occurred only a couple of times a day with strenuous exertion; however, it has started to occur with minimal exertion and is happening more than a dozen times per day. The shortness of breath lasts for less than 5 minutes at a time. He has no cough, chest pressure, chest pain, swelling in his feet, palpitations, orthopnea, or paroxysmal nocturnal dyspnea. Which of the following symptom attributes was not addressed in this description? A) Severity B) Setting in which the symptom occurs C) Timing D) Associated manifestations

A; Ans: A Chapter: 03 Page and Header: 65, The Seven Attributes of a Symptom Feedback: The severity of the symptom was not recorded by the interviewer, so we have no understanding as to how bad the symptom is for this patient. The patient could have been asked to rate his pain on a 0 to 10 scale or used one of the other standardized pain scales available. This allows the comparison of pain intensity before and after an intervention.

9. Mrs. H. comes to your clinic, wanting antibiotics for a sinus infection. When you enter the room, she appears to be very angry. She has a raised tone of voice and states that she has been waiting for the past hour and has to get back to work. She states that she is unimpressed by the reception staff, the nurse, and the clinic in general and wants to know why the office wouldn't call in an antibiotic for her. Which of the following techniques is not useful in helping to calm this patient? A) Avoiding admission that you had a part in provoking her anger because you were late B) Accepting angry feelings from the patient and trying not to get angry in return C) Staying calm D) Keeping your posture relaxed

A; Ans: A Chapter: 03 Page and Header: 75, Adapting Your Interview to Specific Situations Feedback: In this scenario, the provider was 1 hour late in seeing the patient. The provider should acknowledge that he was late and apologize for this, no matter the reason for being late. It often helps to acknowledge that a patient's anger with you is understandable and that you might be angry in a similar situation.

10. A 23-year-old graduate student comes to your clinic for evaluation of a urethral discharge. As the provider, you need to get a sexual history. Which one of the following questions is inappropriate for eliciting the information? A) Are you sexually active? B) When was the last time you had intimate physical contact with someone, and did that contact include sexual intercourse? C) Do you have sex with men, women, or both? D) How many sexual partners have you had in the last 6 months?

A; Ans: A Chapter: 03 Page and Header: 81, Sensitive Topics That Call For Specific Approaches Feedback: This is inappropriate because it is too vague. Given the complaint, you should probably assume that he is sexually active. Sometimes patients may respond to this question with the phrase "No, I just lie there." A specific sexual history will help you to assess this patient's risk for other sexually transmitted infections.

3. A 30-year-old sales clerk comes to your office wanting to lose weight; her BMI is 30.0 kg/m2 . What is the most appropriate amount for a weekly weight reduction goal? A) .5 to 1 pound per week B) 1 to 2.5 pounds per week C) 2.5 to 3.5 pounds per week D) 3.5 to 4.5 pounds per week

A; Ans: A Chapter: 04 Page and Header: 104, Health Promotion and Counseling Feedback: Based on the NIH Obesity Guidelines, this is the weekly weight loss goal to strive for to maintain long-term control of weight. More rapid weight loss than this does not result in a better outcome at one year.

7. A 55-year-old bookkeeper comes to your office for a routine visit. You note that on a previous visit for treatment of contact dermatitis, her blood pressure was elevated. She does not have prior elevated readings and her family history is negative for hypertension. You measure her blood pressure in your office today. Which of the following factors can result in a false high reading? A) Blood pressure cuff is tightly fitted. B) Patient is seated quietly for 10 minutes prior to measurement. C) Blood pressure is measured on a bare arm. D) Patient's arm is resting, supported by your arm at her mid-chest level as you stand to measure the blood pressure.

A; Ans: A Chapter: 04 Page and Header: 114, The Vital Signs Feedback: A blood pressure cuff that is too tightly fitted can result in a false high reading. The other answers are important to observe to obtain an accurate blood pressure reading. JNC-7 also mentions the importance of having the back supported when obtaining blood pressure in the sitting position.

9. An 18-year-old college freshman presents to the clinic for evaluation of gastroenteritis. You measure the patient's temperature and it is 104 degrees Fahrenheit. What type of pulse would you expect to feel during his initial examination? A) Large amplitude, forceful B) Small amplitude, weak C) Normal D) Bigeminal

A; Ans: A Chapter: 04 Page and Header: 114, The Vital Signs Feedback: Fever results in an increased stroke volume, which results in a large-amplitude, forceful pulse. Later in the course of the illness, if dehydration and shock result, you may expect small amplitude and weak pulses.

14. You are seeing an older patient who has not had medical care for many years. Her vital signs taken by your office staff are: T 37.2, HR 78, BP 118/92, and RR 14, and she denies pain. You notice that she has some hypertensive changes in her retinas and you find mild proteinuria on a urine test in your office. You expected the BP to be higher. She is not on any medications. What do you think is causing this BP reading, which doesn't correlate with the other findings? A) It is caused by an "auscultatory gap." B) It is caused by a cuff size error. C) It is caused by the patient's emotional state. D) It is caused by resolution of the process which caused her retinopathy and kidney problems.

A; Ans: A Chapter: 04 Page and Header: 114, The Vital Signs Feedback: The blood pressure is unusual in this case in that the systolic pressure is normal while the diastolic pressure is elevated. Especially with the retinal and urinary findings, you should consider that the BP may be much higher and that an auscultatory gap was missed. This can be avoided by checking for obliteration of the radial pulse while the cuff is inflated. Although a large cuff can cause a slightly lower BP on a patient with a small arm, this does not account for the elevated DBP. Emotional upset usually causes elevation of the BP. Although a process which caused the retinopathy and kidney problems may have resolved, leaving these findings, it is a dangerous assumption that this is the sole cause of the problems seen in this patient.

5. A 27-year-old woman is brought to your office by her mother. The mother tells you that her daughter has been schizophrenic for the last 8 years and is starting to decompensate despite medication. The patient states that she has been taking her antipsychotic and she is doing just fine. Her mother retorts that her daughter has become quite paranoid. When asked why, the mother gives an example about the mailman. She says that her daughter goes and gets the mail every day and then microwaves the letters. The patient agrees that she does this but only because she sees the mailman flipping through the envelopes and she knows he's putting anthrax on the letters. Her mother turns to her and says, "He's only sorting the mail!" Which best describes the patient's abnormality of perception? A) Illusion B) Hallucination C) Fugue state

A; Ans: A Chapter: 05 Page and Header: 145, Techniques of Examination Feedback: An illusion is merely a misinterpretation of real external stimuli. In this case, the mailman is looking through the letters before he puts them in the box. The mother correctly assumes he is sorting the mail but her schizophrenic daughter attributes his actions to being part of a nefarious bioterrorism plot.

14. A nurse is preparing an in-service education program for a group of staff nurses about documentation, including documentation of assessment data. The nurse demonstrates understanding of the significance of documentation by including a discussion of which of the following as playing a role in this area? Select all that apply. A) Joint Commission B) State nurse practice act C) Medicare D) Local or city government E) Institutional agency

A; B; C; E

3. A 75-year-old homemaker brings her 76-year-old husband to your clinic. She states that 4 months ago he had a stroke and ever since she has been frustrated with his problems with communication. They were at a restaurant after church one Sunday when he suddenly became quiet. When she realized something was wrong he was taken to the hospital by EMS. He spent 2 weeks in the hospital with right-sided weakness and difficulty speaking. After hospitalization he was in a rehab center, where he regained the ability to walk and most of the use of his right hand. He also began to speak more, but she says that much of the time "he doesn't make any sense." She gives an example that when she reminded him the car needed to be serviced he told her "I will change the Kool-Aid out of the sink myself with the ludrip." She says that these sayings are becoming frustrating. She wants you to tell her what is wrong and what you can do about it. While you write up a consult to neurology, you describe the syndrome to her. What type of aphasia does he have? A) Wernicke's aphasia B) Broca's aphasia C) Dysarthria

A; Ans: A Chapter: 05 Page and Header: 145, Techniques of Examination Feedback: With Wernicke's aphasia the patient can speak effortlessly and fluently, but his words often make no sense. Words can be malformed or completely invented. Wernicke's area is found on the temporal lobes.

12. You ask a patient to draw a clock. He fills in all the numbers on the right half of the circle. What do you suspect? A) Hemianopsia B) Fatigue C) Oppositional defiant disorder D) Depression

A; Ans: A Chapter: 05 Page and Header: 145, Techniques of Examination Feedback: You should suspect a visual problem because there is no writing on one half of the circle. This is consistent with a hemianopsia, sometimes seen in stroke. These patients may also eat food on only one half of their plate. The other conditions would not account for this pattern.

2. A 72-year-old retired truck driver comes to the clinic with his wife for evaluation of hearing loss. He has noticed some decreased ability to hear what his wife and grandchildren are saying to him. He admits to lip-reading more. He has a history of noise exposure in his young adult years: He worked as a sound engineer at a local arena and had to attend a lot of concerts. Based on this information, what is the most likely finding regarding his hearing acuity? A) Loss of acuity for middle-range sounds B) Increase of acuity for low-range sounds C) Loss of acuity for high-range sounds D) Increase of acuity for high-range sounds

A; Ans: A Chapter: 20 Page and Header: 896, Anatomy and Physiology Feedback: Human speech is considered to be a middle-range sound. During the aging process there is a loss of acuity, starting with high-pitched sounds but extending to the middle range and then into the low range.

16. Which of the following represents age-related changes in the lungs? A) Decrease in chest wall compliance B) Speed of expiration increases C) Increase in respiratory muscle strength D) Increased elastic recoil of lung tissue

A; Ans: A Chapter: 20 Page and Header: 897, Anatomy and Physiology Feedback: The lungs age along with the rest of the body. These changes include decreased lung and chest wall compliance, increased expiratory time, decreased muscle strength and cough, and decreased elastic recoil.

19. Mrs. Buckley is a 75-year-old widow who wants you to look at her teeth because over the past 2 weeks she has had right-sided jaw pain when eating. It does not occur otherwise. She also has had a headache. Which of the following should be considered? A) Palpation of her temples B) Dental referral C) Ultrasound of the gallbladder D) Inquiry about anosmia

A; Ans: A Chapter: 20 Page and Header: 898, Anatomy and Physiology Feedback: This story can be consistent with temporal arteritis, which can cause blindness in 15% of those affected. Early recognition is crucial. Most of these patients will have tenderness over one or both of the temporal arteries, and some have diminished temporal pulses as well. Early treatment with corticosteroids is indicated. It can also be associated with polymyalgia rheumatic, a condition which causes pain in the shoulder girdles and pelvis.

9. Which of the following booster immunizations is recommended in the older adult population? A) Tetanus B) Diphtheria C) Measles D) Mumps

A; Ans: A Chapter: 20 Page and Header: 909, Health Promotion and Counseling Feedback: Older adults who have received the primary series of three tetanus immunizations should receive the single booster dose of tetanus immunization every 10 years.

12. Which of the following brief screening measures is useful in assessing memory? A) Three-item recall B) Serial 7s C) Spelling "world" backward D) Copying intersecting pentagrams

A; Ans: A Chapter: 20 Page and Header: 913, Techniques of Examination Feedback: If the patient is unable to remember three items after 1 minute has passed, then this is a positive screening test and indicates a need for further testing. This is part of the "10-Minute Geriatric Screener."

13. Which of the following questions is part of the screening for physical disability? A) Are you able to go shopping for groceries or clothes? B) Are you able to walk one block? C) Are you able to pass the driver's license test? D) Are you able to perform light dusting and pick up after yourself around the house?

A; Ans: A Chapter: 20 Page and Header: 914, Techniques of Examination Feedback: This is part of the Physical Disability screening portion of the 10-Minute Geriatric Screener.

30. Which of the following is true of assessment of the vascular system in the elderly? A) Fewer than one third of patients with peripheral vascular disease have symptoms of claudication. B) An aortic width of 2.5 cm is abnormal. C) Bruits are commonly benign findings. D) Orthostatic blood pressure and pulse are not useful in this population.

A; Ans: A Chapter: 20 Page and Header: 921, Techniques of Examination Feedback: It is the minority of patients with peripheral vascular disease who experience claudication; therefore, ankle-brachial ratios should be performed more frequently. The aorta should be 3 cm or less. Bruits usually indicate pathology, and even when there is not a significant blockage, the risk of vascular disease throughout the body is increased. Orthostatic vital signs are very useful in this population. Remember to observe the pulse as well, as failure of the heart to increase its rate is a common cause of orthostatic hypotension. This can occur as a result of autonomic neuropathy or medications such as beta- blockers, among other causes.

1. A palliative care nurse is explaining the basis of pain to a group of nurses who provide care on a general medical unit. Which of the following factors would the nurse include? Select all that apply. A) Physiologic B) Psychosocial C) Cutaneous D) Somatic E) Visceral

A; B

16. The nurse is assessing the diet and nutritional status of a client from a different culture. Which of the following questions would be appropriate for the nurse to ask? Select all that apply. A) ìWhat foods do you commonly eat?î B) ìDo you have any special routines for eating?î C) ìAre there any foods that you can't eat?î D) ìDo you eat three meals a day?î E) ìDo you have certain foods to keep you healthy?î

A; B; C; E

1. A nurse has completed the general survey of a client who has been transferred to the unit. The information gathered during the general survey primarily provides the nurse with which of the following? Select all that apply. A) An indication of the level of physical distress experienced by the client B) Clues about the overall health of the client C) A direct link to the client's medical diagnosis D) Indications about normal variations in the status of body systems E) Data relating to the patient's level of social support

A; B; D

23. The gerontologic nurse is using the SPICES screening tool to assess an older adult's health status. The nurse will assess for which of the following health problems? Select all that apply. A) Sleep disturbances B) Infection C) Poor nutrition D) Falls E) Pain

A; C; D

13. A woman brings her 69-year-old husband to the clinic for an evaluation because he has become increasingly forgetful. Which of the following would lead the nurse to suspect that the client has Alzheimer's disease? Select all that apply. A) ìHe repeats the same story, word for word, over and over again.î B) ìHe took a fall when he was replacing a light bulb last month.î C) ìI have to balance the checkbook now because he just won't do it.î D) ìIf I don't tell him when to shower, he won't and will fight me on it.î E) ìHe got lost walking to the pharmacy around the corner the other day.î

A; C; D; E

24. The nurse is evaluating the setting prior to beginning a client's physical examination. The nurse should confirm the presence of which of the following? Select all that apply. A) Adequate lighting B) Cool room temperature C) Quiet surroundings D) Soft chair or table E) Table for equipment F) Door or curtain

A; C; E; F

21. The nurse is reviewing a client's health history and the results of the most recent physical examination. Which of the following data would the nurse identify as being subjective? Select all that apply. A) ìI feel so tired sometimes.î B) Weight: 145 lbs C) Lungs clear to auscultation D) Client complains of a headache E) ìMy father died of a heart attack.î F) Pupils equal, round, and reactive to light

A; D; E

26. The nurse is completing a review of systems for a client. Which of the following information would the nurse document related to the client's musculoskeletal system? Select all that apply. A) Joint stiffness B) Rhinorrhea C) Shortness of breath D) Chest pain E) Muscle strength F) Knee swelling

A; E; F

1. The nurse is preparing to assess the remote memory of a client who has a diagnosis of early stage Alzheimer's disease. Which question would be most appropriate for the nurse to use? A) ìCan you tell me what you have eaten in the last 24 hours?î B) ìWhen did you get your first job?î C) ìWhat did you do last evening?î D) ìHow are an apple and orange the same?î

B

10. A group of students is reviewing material on cultural competence. The students demonstrate understanding of this concept when they identify which of the following as the starting point? A) Cultural awareness B) Cultural desire C) Cultural skill D) Cultural knowledge

B

23. Mrs. Glynn is 90 years old and lives alone. She is able to bathe, dress, prepare her food, and transfer from bed to chair independently. She has children in the area who help her with her medications and transportation needs. Which of the following is considered an instrumental activity of daily living? A) Bathing B) Dressing C) Preparing food D) Transferring from bed to chair

Ans: C Chapter: 20 Page and Header: 906, The Health History Feedback: Instrumental activities of daily living involve higher thought processes such as preparing food, whereas bathing, dressing, and transferring are considered physical activities of daily living.

. Which critical thinking skill helps the nurse see relationships among the data? a. Validation b. Clustering related cues c. Identifying gaps in data d. Distinguishing relevant from irrelevant

B

1. A nurse has completed a comprehensive assessment of a client and has begun the process of data analysis. Data analysis should allow the nurse to produce which of the following direct results? A) Outcomes evaluation B) Nursing diagnoses C) Holistic interventions D) An interdisciplinary plan of care

B

1. A nurse is completing the intake assessment of an older adult who has just relocated to a long-term care facility. Which of the following nursing actions would be most important to ensure accurate data when gathering the resident's information? A) Documenting the data B) Validating the data C) Identifying client support systems D) Determining client needs

B

1. The nurse is assessing a client's psychosocial development in light of Freud's theory. The nurse would interpret the client's status as the outcome of conflict between what variables? A) Cultural norms and personality traits B) Biological desires and social expectations C) Sexual desires and relational desires D) Sociocultural norms and health needs

B

10. A nurse is analyzing the assessment data of a client who has been admitted with exacerbation of heart failure. The nurse has determined that the cue clusters meet the defining characteristics of specific nursing diagnoses. Which of the following would the nurse do next? A) Explain the client's problems to the client and his or her family. B) Verify it with the client and with other health care professionals. C) Validate the diagnosis with the physician. D) Work with the client to begin planning interventions.

B

11. The nurse is assessing an elderly client who is receiving tube feedings via a nasogastric tube. The nurse should assess the client for signs and symptoms of which of the following? A) Gingivitis B) Sinusitis C) Epiglottitis D) Cellulitis

B

12. A nurse is preparing to document conclusions after analyzing data, and he or she includes information about related factors and manifestations. The nurse is formulating which of the following? A) Risk nursing diagnosis B) Actual nursing diagnosis C) Collaborative problem D) Problem for referral

B

12. The nurse is preparing to perform the physical examination of an older adult client who will begin rehabilitation from an ischemic stroke. Which of the following actions would be most appropriate? A) Omit intrusive parts of the exam. B) Try to minimize position changes. C) Allow client to remain dressed. D) Dim the room light to ensure privacy.

B

13. The nurse at a busy primary care clinic is analyzing the data obtained from the following clients. For which clients would the nurse most likely expect to facilitate a referral? A) An 80-year-old client who lives with her daughter B) A 50-year-old client newly diagnosed with diabetes C) An adult presenting for an influenza vaccination D) A teenager seeking information about contraception

B

13. The nurse is applying the principles of Freud's theory of psychosocial development during the health assessment of a young adult client. What assessment question is most likely to elicit data that are meaningful within this theoretical framework? A) ìDo you have a sufficient number of friends?î B) ìDo you have a satisfying sexual relationship?î C) ìHow do you feel about your cultural background?î D) ìDo you consider yourself to be a good person?î

B

13. The nurse is completing the general survey of a client and determines that the client's temperature is 102∞F. Which of the following would the nurse also expect to find? A) Weak, thready pulse B) Heart rate greater than 100 bpm C) Respiratory rate between 12 and 20 breaths/minute D) Diastolic blood pressure 10 mm Hg greater than normal

B

13. The nurse is preparing to assess the peripheral pulses of a client. The nurse should place the client in which position? A) Sitting upright B) Supine C) Sims position D) Prone

B

14. A nurse is modifying an Asian client's diet to accommodate the concept of hot and cold. The nurse demonstrates an understanding of this concept when identifying which of the following as a cold condition? A) Diabetes B) Pneumonia C) Sore throat D) Hypertension

B

14. The nurse is assessing a young adult client in light of Erikson's theory of psychosocial development. During this life stage, what assessment finding would most clearly suggest a lack of successful development? A) The client is dissatisfied with her current job. B) The client describes herself as lonely and isolated. C) The client has been diagnosed with bipolar disorder. D) The client had a child when she was in her late teens.

B

15. A client has been admitted following an unexplained weight loss of 15 pounds over the past 3 months. How should the nurse best assess the subjective component of the client's nutritional status? A) Ask the client to explain MyPlate. B) Obtain a 24-hour diet recall. C) Ask about the contents of one typical meal. D) Elicit the client's favorite foods.

B

15. A nurse has assessed an elderly client and is preparing to analyze the assessment data. Which of the following would the nurse need in order to accurately perform data comparison? A) Client's major complaints B) Client's usual daily patterns C) Client's adherence to treatment D) Client's underlying pathology

B

15. A nurse is obtaining a client's radial pulse. Which of the following actions demonstrates correct technique for this assessment? A) Application of firm pressure on the wrist area along the side of the fifth digit B) Use of two middle fingers lightly applied to wrist area along the thumb side C) Use of the thumb and index finger applied to obliterate the wrist area along the thumb side D) Application of the bell of the stethoscope to the antecubital area of the upper extremity

B

16. A nurse is assessing a client who is exhibiting decorticate posturing. Which of the following would the nurse observe? A) Extended upper extremities B) Internally rotated lower extremities C) Pronated forearms D) Flexed hands at the side of the body

B

16. A nurse is using a nursing minimum data set to document findings following the assessment of a client. This nurse is most likely providing care in which setting? A) Acute care facility B) Long-term care facility C) Urgent care center D) Health clinic

B

17. During an assessment, the nurse determines that a client sees more than one primary care provider and has obtained prescriptions from each provider. Which method would be most appropriate to determine a client's current medication regimen? A) Ask the client to identify which medications taken every day. B) Ask the client to bring all the medications and supplements to an interview. C) Ask the caregiver whether the client is taking prescribed medications. D) Ask the client about the use of any over-the-counter medications.

B

17. The nurse is admitting a client to surgical daycare and is assessing the client's vital signs. When obtaining the client's oral temperature, where should the nurse insert the thermometer? A) At the gum line between the check and tongue B) Deep in the posterior sublingual pocket C) On either side of the frenulum at gingival level D) Just past the teeth, below the tongue

B

17. The nurse observes a client's entire body posture to be somewhat stiff, with his shoulders elevated upward toward the ears. The nurse would most likely interpret this to indicate that the client is experiencing which of the following? A) Confusion B) Anxiety C) Powerlessness D) Restlessness

B

17. When appraising a young adult's psychosocial development within the framework of Erikson's theory, what question should guide the nurse's data collection and analysis? A) Can the client successfully solve problems? B) Has the client successfully achieved intimacy? C) Has the client learned to trust others? D) Can the client teach life skills to others?

B

19. A nurse educator is reviewing the unit's resources about religious groups and their views about blood and blood products, organ donation, and autopsy. A member of which group is most likely to refuse a blood transfusion? A) Christian Scientists B) Jehovah's Witnesses C) Orthodox Jews D) Roman Catholics

B

19. An elderly client with a history of sinusitis has been taking antibiotics for this condition. The nurse should assess for what potential adverse effect of treatment? A) Exacerbation of cardiac dysrhythmias B) Candidal infection C) Overdrying of nasal passages D) Exacerbation of hypertension

B

19. Assessment of an older adult client suggests that the client does not possess formal operational thinking. Within Piaget's framework of development, what nursing diagnosis is the most likely consequence of this developmental deficit? A) Spiritual distress B) Ineffective health maintenance C) Ineffective sexuality pattern D) Risk for suicide

B

19. Due to a change in the client's level of consciousness, a nurse is now assessing a client's temperature by the axillary route. Previously, the client had an oral temperature of 98.4∫F. Which finding would the nurse interpret as corresponding most closely to the client's previous temperature? A) 97.0∫F B) 97.4∫F C) 98.9∫F D) 99.4∫F

B

2. A nurse assesses the skin of an older adult's forearms and observes purpura. The nurse interprets this finding as indicative of which of the following? A) Elder abuse B) Vascular fragility C) Poor circulation D) Herpes zoster

B

20. A client has just been admitted to the postsurgical unit from postanesthetic recovery, and the nurse is in the introductory phase of the client interview. Which of the following activities should the nurse perform first? A) Collaborate with the client to identify problems. B) Explain the purpose of the interview. C) Determine the client's vital signs. D) Obtain family health history data.

B

20. A group of students is reviewing information from class about the purposes of assessment documentation. The students demonstrate understanding of the material when they state which of the following? A) ìDocumentation helps support reimbursement but gives little epidemiologic data.î B) ìDocumentation provides a permanent legal record of care given and not given.î C) ìDocumentation is a viable means of communication but is repetitious.î D) ìDocumentation helps determine client education needs but not staff mix.î

B

20. A nurse asks a client the following question: ìWhat do you do if you have pain?î The nurse is assessing which of the following aspects of cognitive function? A) Orientation B) Judgment C) Abstract reasoning D) Memory

B

20. A nurse in the surgical daycare department has called a client in from the waiting room and is meeting the client for the first time. The nurse immediately observes that the client has a noticeably ìstoopedî posture. How should the nurse best follow up this abnormal assessment finding? A) Facilitate a referral to the hospital's rheumatology department B) Perform a focused assessment of the client's musculoskeletal system C) Obtained a detailed family health history from the client D) Document the assessment finding and inform the anesthesiologist

B

20. A nurse is appraising a colleague's assessment technique as part of a continuing education initiative. The nurse demonstrates the proper technique for light palpation by performing which of the following actions? A) Depressing the skin 1 to 2 centimeters with the dominant hand B) Feeling the surface structures using a circular motion C) Placing the nondominant hand on top of the dominant hand D) Using one hand to apply pressure and the other hand to feel the structure

B

21. A nurse is providing a verbal update to a client's primary care provider because of the client's worsening nausea. When using an SBAR format to provide a report, the nurse should complete the report with which of the following statements? A) ìWhat would you like to do to address this client's nausea?î B) ìI think this client would benefit from an antiemetic.î C) ìThis client has no recent history of any nausea or vomiting.î D) ìThis client rates his nausea as seven out of ten.î

B

21. A nurse is providing care for a client who has hepatic encephalopathy secondary to chronic alcohol abuse. The nurse's assessment reveals that the client often provides incorrect answers to assessment questions. As well, the client makes statements that are not grounded in reality. What nursing diagnosis is suggested by these assessment data? A) Impaired Verbal Communication related to hepatic encephalopathy AMB confusion B) Acute Confusion related to hepatic encephalopathy C) Ineffective Health Maintenance related to alcohol abuse AMB decreased cognition D) Ineffective Coping related to alcohol abuse

B

21. During the interview, the client states, "Is today the 12th? My wife died 2 months ago today." Which of the following responses would be most appropriate? A) "What was the cause of your wife's death?" B) "How does that make you feel right now?" C) "You probably must be sad." D) "Are you feeling sad, depressed, angry, or upset?"

B

21. The nurse is preparing to examine an older adult client. Which of the following would be most appropriate for the nurse to do during the examination? A) Complete the examination as quickly as possible. B) Speak clearly and slowly when explaining a procedure. C) Begin the examination with auscultation instead of inspection. D) Maintain the supine position for each part of the examination.

B

22. An older adult client has been admitted to the intensive care unit after experiencing a serious decline in health due to influenza. The client's family is surprised that influenza could have such serious health consequences. When educating the family about this phenomenon, what should the nurse describe? A) Older adults' immune systems cannot produce new antibodies. B) Older adults have a diminished physiologic reserve. C) Older adults lack resistance to many common viruses. D) Older adults cannot tolerate antibiotics used to treat influenza.

B

23. A nurse is caring for a 70-year-old client from a different culture whose breast cancer has metastasized. The nurse observes that the client tends to defer responsibility for decision making around treatment options to her eldest son. How should the nurse respond to this? A) Explain the disconnect between the client's practice and the principle of client autonomy. B) Confirm that the client wants her son to make decisions and follow those decisions accordingly. C) Attempt to dialogue with the client when her son is not present. D) Refer the family to social work in order to further explore alternative decision- making practices.

B

24. A clinic nurse is conducting a comprehensive assessment of a 70-year-old male client of Native American ethnicity. The nurse observes that the client rarely makes eye contact and holds his head low during the assessment. How should the nurse best interpret this practice? A) The client may not understand the purpose of the assessment. B) The client may be showing the nurse respect. C) The client may be a victim of intimate partner violence. D) The client may not trust the nurse's expertise.

B

24. The nurse is attempting to cluster the data that she collected during the initial assessment of an older adult client. The nurse notes that the client had a swollen left knee and complained of "a bit of soreness" in the joint, but the nurse does not have enough data to support a nursing diagnosis of Impaired Physical Mobility. What should the nurse do next? A) Document a suspected nursing diagnosis of Impaired Physical Mobility. B) Assess the client further for evidence of reduced mobility and decreased range of motion. C) Make a referral to the physical therapist. D) Plan interventions that will conservatively manage the client's joint dysfunction.

B

25. A nurse at an ambulatory clinic is preparing to begin the collection of objective assessment data from a female client. After meeting the client and bringing her into the examination room, what instruction should the nurse provide? A) ìI'll get you to lay down flat on the exam table, please.î B) ìPlease have a seat on the edge of the exam table.î C) ìI'll start the assessment with you standing up and then help you onto the table.î D) ìWhere would you like me to conduct your health assessment?î

B

25. A nurse is attempting to apply the principles of cultural competency in the care of a 72- year-old Asian-American woman who has a spinal cord compression. Which of the following statements should guide the nurse's care? A) The client may view pain as a sign of weak character. B) The client may be reluctant to accept opioids. C) The client may tend to overreport her pain. D) The client may be unable to understand quantitative assessment scales.

B

26. The nurse is assessing an adult client's self-image during the health history interview. What assessment question is most likely to elicit meaningful data? A) ìWhat are the activities that give you the most joy?î B) ìWhat would you describe as your main strengths and weaknesses?î C) ìDo you consider yourself to be a particularly religious person?î D) ìWhat actions are you taking to improve your life?î

B

27. A nurse is admitting a client to the postsurgical unit from the postanesthetic care unit. The nurse has transferred the client from the stretcher to a bed and asked the client if he is experiencing pain. The client acknowledges that he is in pain. What should be the nurse's next action? A) Ask the client to briefly explain his cultural background. B) Assess the client's pain according to COLDSPA. C) Assess the client's self-management skills. D) Assess the client's pain by obtaining a set of vital signs.

B

27. The nurse is reviewing and analyzing data from the initial assessment of a newly admitted client who is a 79-year-old man. What assessment finding most clearly indicates a need for further data? A) The man has male pattern baldness. B) The man has a diffuse rash on his torso. C) The man's heart rate is 63 beats per minute. D) The man had an inguinal hernia repair in 2008.

B

28. A 60-year-old woman with a bunion will undergo surgery later today. The client tells the nurse in the surgical daycare admitting department, "I'm sure I've been asked these questions before. Can't we just focus on my foot and not all these other topics?" How should the nurse best explain the rationale for obtaining a health history? A) "In general, it's necessary for us to gather as much information about each client as possible." B) "We want to make sure your nursing care matches your needs as closely as possible." C) "The care team needs to cross-reference your diagnostic testing with the information that I'm asking you about." D) "We don't want to make the mistake of focusing solely on the medical problem that brought you here."

B

28. A nurse has selected several nursing diagnoses in the process of data analysis of a client with poorly controlled type 1 diabetes. One of these diagnoses is Ineffective Health Maintenance related to infrequent blood glucose monitoring as manifested by elevated HgA1C. The nurse recognizes the need to corroborate this diagnosis with the client. How should the nurse best do this? A) "I think you have a nursing diagnosis of Ineffective Health Maintenance." B) "Would you agree that there's room for improvement in your routines around blood sugar monitoring?" C) "After assessing you, I believe that you're not maintaining your health effectively, specifically around your diabetes." D) "How do you think that you could better maintain your health?"

B

28. A nurse will be working in a clinic in South Asia for several weeks, where the majority of residents have darkly pigmented skin. The nurse should expect a higher-than-average incidence of what integumentary health problem? A) Contact dermatitis B) Vitiligo C) Psoriasis D) Eczema

B

29. A nurse is relying heavily on gestures and simplified language during the assessment of a client from another culture who speaks minimal English. During the lengthy assessment, the nurse asks the client if she is ìokayî by making a circle with his thumb and forefinger. The nurse should be aware of which of the following? A) In some cultures, this gesture denotes confusion. B) In some cultures, this gesture is offensive. C) This gesture has meaning only in American cultures. D) In some cultures, this gesture denotes pain.

B

29. A school nurse is working with kindergarten students. Within Kohlberg's framework of moral development, the nurse should recognize that these students' moral reasoning is primarily motivated by which of the following? A) An innate conscience B) Fear of the negative consequences of individual actions C) Motivation to exhibit behaviors that are culturally normalized D) Adherence to basic moral beliefs

B

29. The nurse is inspecting the dominant hand of an older adult client and notes the presence of irregularly shaped brown lesions on the dorsal surface of the client's hand. What action should the nurse perform next? A) Obtain a tissue sample for pathology B) Compare the appearance of the client's other hand C) Palpate the lesions for tenderness and warmth D) Perform health promotion teaching about sun protection

B

3. A nurse educator is leading a group of nurses in exercises aimed at improving cultural competence. Which of the following would the educator use to best describe an aspect of the term ìcultureî? A) Transmission occurs to another generation through genetics. B) It is shared through norms for behaviors, values, and beliefs. C) It is adapted to a specific environment. D) It is experienced by all people even without human contact.

B

3. The nurse has completed an initial assessment of a newly admitted client and is applying the nursing process to plan the client's care. What principle should the nurse apply when using the nursing process? A) Each step is independent of the others. B) It is ongoing and continuous. C) It is used primarily in acute care settings. D) It involves independent nursing actions.

B

3. When examining the skin of an elderly client, the presence of which skin lesions should indicate a need for referral? A) Cherry angioma B) Actinic keratosis C) Seborrheic keratosis D) Acrochordons

B

30. A client who is new to the facility has a recent history of chronic pain that is attributed to fibromyalgia. The nurse has reviewed the available health records and suspects that pain management will be a major focus of nursing care. How can the nurse best validate this assumption? A) Review the client's medication administration record for analgesic use. B) Ask the client about the most recent experiences of pain. C) Meet with the client's spouse and daughter to discuss the client's pain. D) Collaborate with the physician who is treating the client.

B

30. A nurse is conscientious in adhering to the requirements of the Health Insurance Portability and Accountability Act (HIPAA) when providing care for clients. What action best meets these legal requirements for care? A) Having a colleague audit the nurse's documentation to ensure objectivity B) Maintaining the privacy and confidentiality of clients' medical records C) Using electronic records whenever possible, rather than paper-based records D) Collaborating with the client and his or her family prior to documenting

B

30. An older adult client has been admitted for assessment related to decreased cognition. What assessment finding is most suggestive of delirium as the cause of the client's cognitive changes? A) The client has a family history of cognitive disorders. B) The client recently began a new medication regimen. C) The client has been under significant psychosocial stress. D) The client's cognition has declined over several months.

B

30. An older adult client with osteoarthritis has tearfully admitted to the nurse that she is no longer able to climb the stairs to the second floor of her house due to her knee pain. What nursing diagnosis is suggested by this client's statement? A) Ineffective coping related to knee pain B) Activity intolerance related to knee pain C) Ineffective role performance related to osteoarthritis D) Situational low self-esteem related to osteoarthritis

B

30. The school nurse has learned that a 14-year-old student is having social difficulties. According to Erikson, what is the most likely source of this child's stress? A) The student is experiencing moral dilemmas. B) The student is having difficulty creating an identity. C) The student is experiencing a sexual crisis. D) The student having difficulty understanding the viewpoints of others.

B

4. A client has received a diagnosis of chronic nonmalignant pain. The nurse who is planning this client's nursing care should understand that this client has experienced this pain for at least how many months? A) 3 B) 6 C) 9 D) 12

B

4. A nurse is reviewing a colleague's documentation of a client assessment. The nurse reads that the client's radial pulse was 2+. How should the nurse interpret this assessment finding? A) The client's radial pulse occluded easily. B) The client's radial pulse occluded with moderate pressure. C) The client's radial pulse occluded with very firm pressure. D) The client's radial pulse could not be manually occluded.

B

4. The nurse is using a Wood's light for a client who has complaints of itching, burning, and peeling of the skin between his toes. The nurse is assessing for what etiology of the client's symptoms? A) Parasitic infection B) Fungal infection C) Bacterial infection D) Allergic reaction

B

6. In response to a client's query, the nurse is explaining the differences between the physician's medical exam and the comprehensive health assessment performed by the nurse. The nurse should describe the fact that the nursing assessment focuses on which aspect of the client's situation? A) Current physiologic status B) Effect of health on functional status C) Past medical history D) Motivation for adherence to treatment

B

7. A nurse practitioner is performing a comprehensive physical examination of a 51-year- old man. After performing a digital-rectal exam for prostate enlargement and tenderness, the nurse checks the fecal material on the gloved finger for the presence of which of the following? A) Parasites B) Blood C) Bacteria D) Fungus

B

7. A nursing student has been assigned to the care of a client whose history suggests the need for a mental status assessment. This client most likely has a history of health problems affecting what body system? A) Respiratory B) Neurologic C) Cardiovascular D) Renal

B

7. An African-American woman collapses at the funeral of her mother and later states that she could hear everything people were saying to her but, for a brief period, she could not move. The nurse interprets this as which of the following? A) Spell B) Falling out C) Empacho D) Susto

B

7. The nurse is working with an older adult client and is attempting to determine whether the client deems her life to have been meaningful and valuable. As well, the nurse has addressed the client's acceptance of the inevitability of death. This nurse's actions are best understood within the ideas of which theorist? A) Freud B) Erikson C) Piaget D) Kohlberg

B

8. The nurse has begun a client's assessment and is applying the blood pressure cuff on a client's arm. Which action would be most appropriate? A) The cuff is wrapped loosely around the arm. B) The cuff is placed about 1 inch above the antecubital area. C) The bladder inside the cuff encircles 50% of the arm circumference. D) The nurse can fit three to four fingers under the inflated cuff.

B

8. The nurse is examining an older adult client and using a goniometer. Which of the following would the nurse be assessing? A) Extremity edema B) Joint flexion/extension C) Two-point discrimination D) Vibratory sensation

B

9. A client's recent episode of becoming lost near his home has prompted the nurse to use the Saint Louis University Mental Status (SLUMS) Assessment Tool. The nurse should begin this assessment by asking what question? A) ìHow would you respond if someone said that you might have dementia?î B) ìCan I ask you some questions about your memory?î C) ìDo you generally consider yourself to be an intelligent person?î D) ìI want to ask you some questions to see if you have Alzheimer's.î

B

9. A nurse assesses a client's blood pressure and the findings suggest orthostatic hypotension. Which area should the nurse emphasize during client education? A) Daily exercise routine B) Prevention of falls C) Diet high in iron D) Vitamin supplementation

B

9. Based on a colleague's feedback, a nurse learns that she is aware of cultural differences in a general way but does not know what the specific differences are or how to communicate with a person of a specific culture. This nurse exhibits which of the following? A) Unconscious incompetence B) Conscious incompetence C) Conscious competence D) Unconscious competence

B

The nurse is conducting a class for new graduate nurses. During the teaching session, the nurse should keep in mind that novice nurses, without a background of skills and experience from which to draw, are more likely to make their decisions using: a. Intuition. b. A set of rules. c.Articles in journals. d. Advice from supervisors

B

1. The nurse is interviewing a female Hispanic client who is scheduled for a cardiovascular education program. The client states, ìI can't eat and I don't sleep because my daughter left to return to Mexico. I am sad and nervous. I need rest.î The nurse suspects that she is suffering from susto. Which action by the nurse would be best? A) Give her a multivitamin supplement. B) Encourage her to exercise. C) Reschedule the education program. D) Refer her to a counselor.

C

2. The components of the health history include all of the following except which one? A) Review of systems B) Thorax and lungs C) Present illness D) Personal and social items

B; Ans: B Chapter: 01 Page and Header: 4, Patient Assessment: Comprehensive or Focused Feedback: The thorax and lungs are part of the physical examination, not part of the health history. The others answers are all part of a complete health history.

4. Is the following information subjective or objective? Mr. M. has a respiratory rate of 32 and a pulse rate of 120. A) Subjective B) Objective

B; Ans: B Chapter: 01 Page and Header: 6, Differences Between Subjective and Objective Data Feedback: This is a measurement obtained by the examiner, so it is considered objective data. The patient is unlikely to be able to give this information to the examiner.

5. The following information is recorded in the health history: "The patient has had abdominal pain for 1 week. The pain lasts for 30 minutes at a time; it comes and goes. The severity is 7 to 9 on a scale of 1 to 10. It is accompanied by nausea and vomiting. It is located in the mid-epigastric area." Which of these categories does it belong to? A) Chief complaint B) Present illness C) Personal and social history D) Review of systems

B; Ans: B Chapter: 01 Page and Header: 6, The Comprehensive Adult Health History Feedback: This information describes the problem of abdominal pain, which is the present illness. The interviewer has obtained the location, timing, severity, and associated manifestations of the pain. The interviewer will still need to obtain information concerning the quality of the pain, the setting in which it occurred, and the factors that aggravate and alleviate the pain. You will notice that it does include portions of the pertinent review of systems, but because it relates directly to the complaint, it is included in the history of present illness.

9. The following information is best placed in which category? "The patient has had three cesarean sections." A) Adult illnesses B) Surgeries C) Obstetrics/gynecology D) Psychiatric

B; Ans: B Chapter: 01 Page and Header: 9, Past History Feedback: A cesarean section is a surgical procedure. Approximate dates or the age of the patient at the time of the surgery should also be recorded.

4. A 22-year-old advertising copywriter presents for evaluation of joint pain. The pain is new, located in the wrists and fingers bilaterally, with some subjective fever. The patient denies a rash; she also denies recent travel or camping activities. She has a family history significant for rheumatoid arthritis. Based on this information, which of the following pathologic processes would be the most correct? A) Infectious B) Inflammatory C) Hematologic D) Traumatic

B; Ans: B Chapter: 02 Page and Header: 27, Assessment and Plan: The Process of Clinical Reasoning Feedback: The description is most consistent with an inflammatory process, although all the other etiologies should be considered. Lyme disease is an infection which commonly causes arthritis, hemophilia is a hematologic condition which can cause bleeding in the joints, and trauma can obviously cause joint pain. Your clinical reasoning skills are important for sorting through all of the data to arrive at the most likely conclusion.

13. On the way to see your next patient, you glance at the calendar and make a mental note to buy a Mother's Day card. Your patient is Ms. Hernandez, a 76-year-old widow who lost her husband in May, two years ago. She comes in today with a headaches, abdominal pain, and general malaise. This happened once before, about a year ago, according to your detailed office notes. You have done a thorough evaluation but are unable to arrive at a consistent picture to tie these symptoms together. This is an example of a: A) Pathophysiologic problem B) Psychopathologic problem

B; Ans: B Chapter: 02 Page and Header: 38, The Challenges of Clinical Data Feedback: It is not uncommon for patients to experience psychopathologic symptoms around the anniversary of a traumatic event. The time of year and the lack of an obvious connection between Ms. Hernandez's symptoms would make you consider this as a possibility. You will note that although this might have been an early consideration in your hypothesis generation, it is key to convince yourself that there is not a physiologic explanation for these symptoms, by performing a careful history and examination.

9. A 55-year-old data entry operator comes to the clinic to establish care. She has the following symptoms: headache, neck pain, sinus congestion, sore throat, ringing in ears, sharp brief chest pains at rest, burning abdominal pain with spicy foods, constipation, urinary frequency that is worse with coughing and sneezing, and swelling in legs. This cluster of symptoms is explained by: A) One disease process B) More than one disease process

B; Ans: B Chapter: 02 Page and Header: 38, The Challenges of Clinical Data Feedback: The patient appears to have several possible conditions: allergic rhinitis, arthritis, conductive hearing loss, pleuritic chest pains, heartburn, stress urinary incontinence, and venous stasis, among other conditions. Although we always try, it is very difficult to assign all of these symptoms to one cohesive diagnosis.

3. Alexandra is a 28-year-old editor who presents to the clinic with abdominal pain. The pain is a dull ache, located in the right upper quadrant, that she rates as a 3 at the least and an 8 at the worst. The pain started a few weeks ago, it lasts for 2 to 3 hours at a time, it comes and goes, and it seems to be worse a couple of hours after eating. She has noticed that it starts after eating greasy foods, so she has cut down on these as much as she can. Initially it occurred once a week, but now it is occurring every other day. Nothing makes it better. From this description, which of the seven attributes of a symptom has been omitted? A) Setting in which the symptom occurs B) Associated manifestations C) Quality D) Timing

B; Ans: B Chapter: 03 Page and Header: 65, The Seven Attributes of a Symptom Feedback: The interviewer has not recorded whether or not the pain has been accompanied by nausea, vomiting, fever, chills, weight loss, and so on. Associated manifestations are additional symptoms that may accompany the initial chief complaint and that help the examiner to start refining his or her differential diagnosis.

5. You are interviewing an elderly woman in the ambulatory setting and trying to get more information about her urinary symptoms. Which of the following techniques is not a component of adaptive questioning? A) Directed questioning: starting with the general and proceeding to the specific in a manner that does not make the patient give a yes/no answer B) Reassuring the patient that the urinary symptoms are benign and that she doesn't need to worry about it being a sign of cancer C) Offering the patient multiple choices in order to clarify the character of the urinary symptoms that she is experiencing D) Asking her to tell you exactly what she means when she states that she has a urinary tract infection

B; Ans: B Chapter: 03 Page and Header: 68, Building a Therapeutic Relationship: The Techniques of Skilled Interviewing Feedback: Reassurance is not part of clarifying the patient's story; it is part of establishing rapport and empathizing with the patient.

20. When using an interpreter to facilitate an interview, where should the interpreter be positioned? A) Behind you, the examiner, so that the lips of the patient and the patient's nonverbal cues can be seen B) Next to the patient, so the examiner can maintain eye contact and observe the nonverbal cues of the patient C) Between you and the patient so all parties can make the necessary observations D) In a corner of the room so as to provide minimal distraction to the interview

B; Ans: B Chapter: 03 Page and Header: 75, Adapting Your Interview to Specific Situations Feedback: Interpreters are invaluable in encounters where the examiner and patient do not speak the same language, including encounters with the deaf. It should be noted that deaf people from different regions of the world use different sign languages. The priority is for you to have a good view of the patient. Remember to use short, simple phrases while speaking directly to the patient and ask the patient to repeat back what he or she understands.

8. Mrs. T. comes for her regular visit to the clinic. She is on your schedule because her regular provider is on vacation and she wanted to be seen. You have heard about her many times from your colleague and are aware that she is a very talkative person. Which of the following is a helpful technique to improve the quality of the interview for both the provider and the patient? A) Allow the patient to speak uninterrupted for the duration of the appointment. B) Briefly summarize what you heard from the patient in the first 5 minutes and then try to have her focus on one aspect of what she told you. C) Set the time limit at the beginning of the interview and stick with it, no matter what occurs in the course of the interview. D) Allow your impatience to show so that the patient picks up on your nonverbal cue that the appointment needs to end.

B; Ans: B Chapter: 03 Page and Header: 75, Adapting Your Interview to Specific Situations Feedback: You can also say, "I want to make sure I take good care of this problem because it is very important. We may need to talk about the others at the next appointment. Is that okay with you?" This is a technique that can help you to change the subject but, at the same time, validate the patient's concerns; it also can provide more structure to the interview.

11. Mr. Q. is a 45-year-old salesman who comes to your office for evaluation of fatigue. He has come to the office many times in the past with a variety of injuries, and you suspect that he has a problem with alcohol. Which one of the following questions will be most helpful in diagnosing this problem? A) You are an alcoholic, aren't you? B) When was your last drink? C) Do you drink 2 to 3 beers every weekend? D) Do you drink alcohol when you are supposed to be working?

B; Ans: B Chapter: 03 Page and Header: 81, Sensitive Topics That Call for Specific Approaches Feedback: This is a good opening question that is general and neutral in tone; depending on the timing, you will be able to ask for more specific information related to the patient's last drink. The others will tend to stifle the conversation because they are closed-ended questions. Answer D implies negative behavior and may also keep the person from sharing freely with you.

1. A 15-year-old high school sophomore and her mother come to your clinic because the mother is concerned about her daughter's weight. You measure her daughter's height and weight and obtain a BMI of 19.5 kg/m2 . Based on this information, which of the following is appropriate? A) Refer the patient to a nutritionist and a psychologist because the patient is anorexic. B) Reassure the mother that this is a normal body weight. C) Give the patient information about exercise because the patient is obese. D) Give the patient information concerning reduction of fat and cholesterol in her diet because she is obese.

B; Ans: B Chapter: 04 Page and Header: 104, Health Promotion and Counseling Feedback: The patient has a normal BMI; the range for a normal BMI is 18.5 to 24.9 kg/m2 . You may be able to give the patient and her mother the lower limit of normal in pounds for her daughter's height, or instruct her in how to use a BMI table.

8. A 49-year-old truck driver comes to the emergency room for shortness of breath and swelling in his ankles. He is diagnosed with congestive heart failure and admitted to the hospital. You are the student assigned to do the patient's complete history and physical examination. When you palpate the pulse, what do you expect to feel? A) Large amplitude, forceful B) Small amplitude, weak C) Normal D) Bigeminal

B; Ans: B Chapter: 04 Page and Header: 114, The Vital Signs Feedback: Congestive heart failure is characterized by decreased stroke volume or increased peripheral vascular resistance, which would result in a small-amplitude, weak pulse. Subtle differences in amplitude are usually best detected in large arteries close to the heart, like the carotid pulse. You may not be able to notice these in other locations.

16. You are observing a patient with heart failure and notice that there are pauses in his breathing. On closer examination, you notice that after the pauses the patient takes progressively deeper breaths and then progressively shallower breaths, which are followed by another apneic spell. The patient is not in any distress. You make the diagnosis of: A) Ataxic (Biot's) breathing B) Cheyne-Stokes respiration C) Kussmaul's respiration D) COPD with prolonged expiration

B; Ans: B Chapter: 04 Page and Header: 119, Respiratory Rate and Rhythm Feedback: Cheyne-Stokes respiration can be seen in patients with heart failure and is usually not a sign of an immediate problem. Ataxic breathing is very irregular in rhythm and depth and is seen with brain injury. Kussmaul's respiration is seen in patients with a metabolic acidosis, as they are trying to rid their bodies of carbon dioxide to compensate. Respirations in COPD are usually regular and are not usually associated with apneic episodes.

17. Mr. Garcia comes to your office for a rash on his chest associated with a burning pain. Even a light touch causes this burning sensation to worsen. On examination, you note a rash with small blisters (vesicles) on a background of reddened skin. The rash overlies an entire rib on his right side. What type of pain is this? A) Idiopathic pain B) Neuropathic pain C) Nociceptive or somatic pain D) Psychogenic pain

B; Ans: B Chapter: 04 Page and Header: 121, Acute and Chronic Pain Feedback: This vignette is consistent with a diagnosis of herpes zoster, or shingles. This is caused by reemergence of dormant varicella (chickenpox) viruses from Mr. Garcia's nerve root. The characteristic burning quality without a history of an actual burn makes one think of neuropathic pain. It will most likely remain for months after the rash has resolved. There is no evidence of physical injury and this is a peculiar distribution, making nociceptive pain less likely. There is no evidence of a psychogenic etiology for this, and the presence of a rash makes this possibility less likely as well. Because of your astute diagnostic abilities, the pain is not idiopathic.

6. A 22-year-old man is brought to your office by his father to discuss his son's mental health disorder. The patient was diagnosed with schizophrenia 6 months ago and has been taking medication since. The father states that his son's dose isn't high enough and you need to raise it. He states that his son has been hearing things that don't exist. You ask the young man what is going on and he tells you that his father is just jealous because his sister talks only to him. His father turns to him and says, "Son, you know your sister died 2 years ago!" His son replies "Well, she still talks to me in my head all the time!" Which best describes this patient's abnormality of perception? A) Illusion B) Hallucination C) Fugue state

B; Ans: B Chapter: 05 Page and Header: 145, Techniques of Examination Feedback: A hallucination is a subjective sensory perception in the absence of real external stimuli. The patient can hear, see, smell, taste, or feel something that does not exist in reality. In this case, his sister has passed away and cannot be speaking to him, although in his mind he can hear her. This is an example of an auditory hallucination, but hallucinations can occur with any of the five senses.

9. A 72-year-old African-American male is brought to your clinic by his daughter for a follow-up visit after his recent hospitalization. He had been admitted to the local hospital for speech problems and weakness in his right arm and leg. On admission his MRI showed a small stroke. The patient was in rehab for 1 month following his initial presentation. He is now walking with a walker and has good use of his arm. His daughter complains, however, that everyone is still having trouble communicating with the patient. You ask the patient how he thinks he is doing. Although it is hard for you to make out his words you believe his answer is "well . . . fine . . . doing . . . okay." His prior medical history involved high blood pressure and coronary artery disease. He is a widower and retired handyman. He has three children who are healthy. He denies tobacco, alcohol, or drug use. He has no other current symptoms. On examination he is in no acute distress but does seem embarrassed when it takes him so long to answer. His blood pressure is 150/90 and his other vital signs are normal. Other than his weak right arm and leg his physical examination is unremarkable. What disorder of speech does he have? A) Wernicke's aphasia B) Broca's aphasia C) Dysarthria

B; Ans: B Chapter: 05 Page and Header: 145, Techniques of Examination Feedback: In Broca's aphasia patients articulate very slowly and with a great deal of effort. Nouns, verbs, and important adjectives are usually present and only small grammatical words are dropped from speech. Broca's area is on the lateral portion of the frontal lobes.

15. A 29-year-old woman comes to your office. As you take the history, you notice that she is speaking very quickly, and jumping from topic to topic so rapidly that you have trouble following her. You are able to find some connections between ideas, but it is difficult. Which word describes this thought process? A) Derailment B) Flight of ideas C) Circumstantiality D) Incoherence

B; Ans: B Chapter: 05 Page and Header: 145, Techniques of Examination Feedback: This represents flight of ideas because the ideas are connected in some logical way. Derailment, or loosening of associations, has more disconnection within clauses. Circumstantiality is characterized by the patient speaking "around" the subject and using excessive detail, though thoughts are meaningfully connected. Incoherence lacks meaningful connection and often has odd grammar or word use. Although severe flight of ideas can produce this condition, evidence is not present in this vignette.

8. A 23-year-old ticket agent is brought in by her husband because he is concerned about her recent behavior. He states that for the last 2 weeks she has been completely out of control. He says that she hasn't showered in days, stays awake most of the night cleaning their apartment, and has run up over $1,000 on their credit cards. While he is talking, the patient interrupts him frequently and declares this is all untrue and she has never been so happy and fulfilled in her whole life. She speaks very quickly, changing the subject often. After a longer than normal interview you find out she has had no recent illnesses or injuries. Her past medical history is unremarkable. Both her parents are healthy but the husband has heard rumors about an aunt with similar symptoms. She and her husband have no children. She smokes one pack of cigarettes a day (although she has been chain-smoking in the last 2 weeks), drinks four to six drinks a week, and smokes marijuana occasionally. On examination she is very loud and outspoken. Her physical examination is unremarkable. Which mood disorder does she most likely have? A) Major depressive episode B) Manic episode C) Dysthymic disorder

B; Ans: B Chapter: 05 Page and Header: 160, Table 5-2 Feedback: Mania consists of a persistently elevated mood for at least 1 week with symptoms such as inflated self-esteem, decreased need for sleep, pressured speech, racing thoughts, and involvement in high-risk activities (such as drug use, spending sprees, and indiscriminate sexual activity). In this case, the patient has racing thoughts and pressured speech, has a decreased need for sleep, and is engaging in high- risk activities (spending sprees).

10. A 35-year-old stockbroker comes to your office, complaining of feeling tired and irritable. She also says she feels like nothing ever goes her way and that nothing good ever happens. When you ask her how long she has felt this way she laughs and says, "Since when have I not?" She relates that she has felt pessimistic about life in general since she was in high school. She denies any problems with sleep, appetite, or concentration, and states she hasn't thought about killing herself. She reports no recent illnesses or injuries. She is single. She smokes one pack of cigarettes a day, drinks occasionally, and hasn't taken any illegal drugs since college. Her mother suffers from depression and her father has high blood pressure. On examination her vital signs and physical examination are unremarkable. What mental health disorder best describes her symptoms? A) Major depressive episode B) Dysthymic disorder C) Cyclothymic disorder

B; Ans: B Chapter: 05 Page and Header: 160, Table 5-2 Feedback: Someone with dysthymia has a depressed mood and symptoms for most of the day, more days than not, for at least 2 years. The disorder generally begins in adolescence and is fairly stable throughout life. Although the symptoms are similar to those of major depression (in this case, fatigue and irritability), they are milder and fewer.

18. Mr. Chin is an 82-year-old man who comes to your office for a routine check. On examination, you notice a somewhat high-pitched murmur in the second right intercostal space during systole. It does not radiate and the rest of his examination is normal for his age. Which is true of the most likely cause of this murmur? A) It often decreases carotid upstroke. B) It carries with it increased risk for cardiovascular disease. C) It is usually accompanied by an S3 gallop. D) It is found in 10% of otherwise normal elderly patients.

B; Ans: B Chapter: 20 Page and Header: 898, Anatomy and Physiology Feedback: This murmur most likely represents aortic sclerosis, a common murmur affecting about one third of those near 60 years of age. It is caused by calcification of the valve and is associated with cardiovascular risk. Aortic sclerosis does not usually cause obstruction to normal flow, so carotid upstroke should be normal, and it is not associated with an S3 gallop.

6. A 78-year-old retired seamstress comes to the office for a routine check-up. You obtain an ECG (electrocardiogram) because of her history of hypertension. You diagnose a previous myocardial infarction and ask her if she had any symptoms related to this. Which of the following symptoms would be more common in this patient's age group for an acute myocardial infarction? A) Chest pain B) Syncope C) Pain radiating into the left arm D) Pain radiating into the jaw

B; Ans: B Chapter: 20 Page and Header: 903, The Health History Feedback: This is an atypical symptom and more likely to be seen in this patient's age group.

25. Mr. White's son brings him in today because he notes that Mr. White has not been himself lately. He seems forgetful and has not taken care of himself as he normally does. He has reported falling twice at home to his son and has telephoned late at night because of insomnia. His blood pressure and diabetes have been difficult to control and his warfarin dosing has become more difficult. Which of the following should you suspect? A) Alzheimer's dementia B) Alcohol use C) Urinary tract infection D) Stroke

B; Ans: B Chapter: 20 Page and Header: 908, The Health History Feedback: All of these answers are common diseases of the elderly and many have atypical presentations in this age group. The fact that his hypertension has become more difficult to control and his warfarin dosing is challenging to manage should lead you to consider that there is alcohol use. Further questioning, quantifying his use of alcohol, and application of the CAGE questionnaire may be useful.

11. A 73-year-old retired accountant comes to your office for her annual examination. She has incontinence of urine when she coughs or sneezes. She takes several medications for control of hypertension and diabetes. You use the DIAPERS mnemonic to assess the cause of her incontinence. All of the following are items represented by the mnemonic except for: A) Atrophic vaginitis B) Depression C) Pharmaceuticals D) Restricted mobility

B; Ans: B Chapter: 20 Page and Header: 913, Techniques of Examination Feedback: Depression is not a risk factor for incontinence. The D in the mnemonic stands for delirium.

10. A nurse has completed gathering some basic data about a client who has multiple health problems that stem from heavy alcohol use. The nurse has then reflected on her personal feelings about the client and his circumstances. The nurse does this primarily to accomplish which of the following? A) Determine if pertinent data has been omitted B) Identify the need for referral C) Avoid biases and judgments D) Construct a plan of care

C

10. A task force has been established at a hospital with the aim of overhauling the assessment forms that are used throughout the facility. Which of the following options is most likely to help standardize the process of data collection? A) Open-ended form B) Integrated cued checklist form C) Cued or checklist form D) Nursing minimum data set

C

10. Assessment of a client who has suffered a recent stroke reveals that he is unresponsive to all stimuli and his eyes remain closed. The nurse documents the client's level of consciousness as which of the following? A) Obtunded B) Stupor C) Coma D) Lethargy

C

10. The children of an elderly client tell the nurse, ìHe has lost his appetite. He eats very small amounts, and only twice a day.î Which suggestion would be most appropriate? A) Inform them that he will eat when he is hungry. B) Counsel them to weigh him daily. C) Recommend nutrient-dense foods. D) Advise them to restrict fluid intake.

C

10. The nurse is conducting a health interview and has asked the client, ìHow would you describe yourself to others?î The client's response informs the nurse's assessment of which of the following? A) The client's morality and honesty B) The client's aspirations C) The client's self-concept D) The client's superego

C

23. The nurse is gathering the necessary equipment preparatory to examining a client's ears. The nurse will be checking bone and air conduction of sound. Which of the following should the nurse obtain? A) Penlight B) Tongue depressor C) Tuning fork D) Otoscope

C

11. A nurse's data analysis has led to the formulation of a risk nursing diagnosis. Which of the following best demonstrates accurate documentation of a risk nursing diagnosis? A) Risk for fatigue related to increased job demands, as manifested by feelings of exhaustion and frequent naps B) Risk for infection, as manifested by lack of client knowledge of wound care C) Risk for violence related to history of overt, aggressive acts D) Risk for altered respiratory function related to environmental allergens, as manifested by asthma

C

11. During the health interview of a new client, the nurse has explored the client's decision- making strategies. These data are most essential to the developmental theory of which theorist? A) Freud B) Kohlberg C) Piaget D) Erikson

C

11. The nurse is collecting data from a client who has recently been diagnosed with type 1 diabetes and who will begin an educational program. The nurse is collecting subjective and objective data. Which of the following would the nurse categorize as objective data? A) Family history B) Occupation C) Appearance D) History of present health concern

C

12. A client rates his pain as 9 on a scale of 1 to 10. The nurse would expect to assess which of the following? A) Constricted pupils B) Hypotension C) Increased serum glucose D) Flaccid muscles

C

12. A clinic nurse has reviewed a new client's available health record and will now begin taking the client's health history. Which of the following questions should the nurse ask first when obtaining the health history? A) "Do you have adequate health insurance coverage?" B) "Are you generally fairly healthy?" C) "What is your major health concern at this time?" D) "Did you bring all your medications with you?"

C

12. The advanced practice nurse is preparing to perform a pelvic examination on an elderly female client. Which of the following would the nurse expect to find? A) Elongation of the vagina B) Thick, pale epithelium C) Decreased vaginal secretions D) Palpable ovaries

C

12. The nurse attends a Native-American Alcoholic Anonymous support group and develops close relationships with three group members. The nurse is demonstrating which of the following? A) Cultural desire B) Cultural awareness C) Cultural encounter D) Cultural knowledge

C

14. A client has questioned why the nurse asked him how his family members usually treat their pain. Which of the following would be the most appropriate response by the nurse? A) ìIt is just a way for me to more fully understand you and your upbringing.î B) ìIt helps me to direct interventions toward your cultural history.î C) ìIt helps me to determine how the family understands and perceives pain.î D) ìIt will allow me to see if you are more likely to react to pain in a negative manner.î

C

14. As part of a mental status assessment, the nurse asks a client to draw the face of a clock. This will allow the nurse to assess which of the following domains of mental status? A) Concentration and orientation B) Perceptions and thought processes C) Visual perceptual and constructional ability D) Expressions and feelings

C

14. The nurse has collected objective and subjective data during the assessment of a client who has been admitted for the treatment of an exacerbation of chronic obstructive pulmonary disease (COPD). During the current phase of the diagnostic reasoning process, the nurse is writing down thoughts about each cue cluster of data that was collected. The nurse is involved in which step of the diagnostic reasoning process? A) Step One: Identify Abnormal Data and Strengths B) Step Two: Cluster Data C) Step Three: Draw Inferences D) Step Four: Propose Possible Nursing Diagnoses

C

14. When assessing the temperature of the feet of an older client with diabetes, the nurse would use which part of the hand to obtain the most accurate assessment data? A) Finger pad surface B) Palmar hand surface C) Dorsal hand surface D) Ulnar hand surface

C

15. A nurse has completed an assessment of a client with cholecystitis and is about to document the findings. Which statement best reflects accurate documentation? A) Client appears upset about upcoming surgery. B) Client was interviewed about previous history of hypertension. C) Skin pale, warm, and dry without evidence of lesions. D) Client's oral intake is satisfactory.

C

15. When reviewing cultural differences that relate to the incidence and prevalence of disease among various cultural groups, the nurse would expect to see the highest prevalence of asthma in which group? A) Non-Hispanic blacks B) Caucasians C) African Americans D) Southeast Asians

C

16. A client's elevated body mass index (BMI) has prompted the nurse to assess the client's activity and exercise level. Which statement would indicate to the nurse that the client is getting the recommended amount of exercise? A) "I walk briskly on the treadmill once or twice a week." B) "I play basketball with a team every Friday night without fail." C) "I go to a step class for an hour three times a week." D) "I swim for at least half an hour each Saturday morning."

C

16. A nurse is applying the diagnostic reasoning process in the care of a client with a number of comorbidities. Which of the following descriptions best characterizes Step Two, Clustering Data? A) Hypothesizing of any potentially applicable health promotion diagnoses, risk diagnoses, and actual diagnoses B) Documentation of all professional judgments along with any data that support those judgments C) Examining identified abnormal findings and strengths for cues that are related D) Evaluation of both subjective and objective data to identify strengths and abnormal findings

C

16. The nurse is assessing the skin condition and color of an African-American client. Which of the following would the nurse document as an abnormal finding? A) Evenly distributed color B) Light to medium dark brown skin C) Ashen gray skin color D) Lack of visible pores

C

17. A nurse has documented the findings of a comprehensive assessment of a new client. What is the primary rationale that the nurse should identify for accurate and thorough documentation? A) Guaranteeing a continual assessment process B) Identifying abnormal data C) Assuring valid conclusions from analyzed data D) Allowing for drawing inferences and identifying problems

C

17. A nurse is creating a concept map of the pathophysiology of pain. The nurse should identify which of the following as being responsible for transmitting pain sensations to the central nervous system? A) Transduction B) Modulation C) Nociceptors D) Cytokines

C

17. An experienced nurse is teaching a recently graduated colleague about common pitfalls encountered in the diagnostic reasoning process. The experienced nurse should identify a need for further teaching if the new graduate identifies which of the following as a pitfall? A) View of things as either right or wrong B) Overemphasis on details C) Inclusion of valid data D) Clustering of unrelated cues

C

18. A client who has fractured her arm is describing her pain as ìexcruciating.î The nurse determines that the client is most likely experiencing what type of pain? A) Cutaneous B) Visceral C) Deep somatic D) Radiating

C

18. A nurse has received a report on a client who will soon be admitted to the medical unit from the emergency department. When preparing for the assessment phase of the nursing process, which of the following should the nurse do first? A) Collect objective data. B) Validate important data. C) Collect subjective data. D) Document the data.

C

18. A nurse is reviewing a depression questionnaire completed by a client. Which of the following would the nurse interpret as being suggestive of depression? A) ìOccasionally I feel like my attention wanders.î B) ìI haven't noticed any change in my appetite.î C) ìIt usually takes me over an hour to fall asleep.î D) ìI might wake up once during the night but not often.î

C

18. An elderly client's history reveals the use of antihistamines. When inspecting the client's mouth, which of the following would the nurse expect to find? A) Resorption of the gum ridge B) Swollen, red tongue C) Decreased saliva production D) Pocketing of food

C

18. An instructor is describing various ways that a nurse can validate data to a group of nursing students. The instructor determines that additional teaching is necessary when the students identify which of the following as a reliable method? A) Repeating the assessment B) Asking additional questions C) Having the client repeat what was said D) Checking findings with another health care professional

C

18. An instructor is teaching a student about the proper use of a stethoscope. The instructor determines the need for additional teaching when the student states which of the following? A) ìPlastic tubing should be longer than 3 feet.î B) ìThe bell is used after using the diaphragm.î C) ìWhen using the bell, push on it lightly.î D) ìA diaphragm picks up low-pitched sounds.î

C

18. An older adult client has been admitted to the medical unit after suffering an exacerbation of chronic obstructive pulmonary disease (COPD). Which of the following should the nurse do to assess the depth of the client's respirations? A) Count the respirations for 30 seconds and multiply by 2. B) Place the client's arm across the chest while palpating the pulse. C) Observe the client's chest expansion bilaterally. D) Percuss the client's posterior thorax

C

19. A gerontologic nurse is assessing the speech of an older adult client. Which of the following would the nurse characterize as an expected assessment finding? A) Repetition B) Rapid speech C) Moderate pace D) Loud tone

C

19. A nurse has identified a goal of developing his critical thinking skills. In order to facilitate this goal, what action should the nurse prioritize? A) Applying quick decision-making B) Seeking new experiences C) Maintaining an open mind D) Maintaining a stable and static knowledge base

C

2. A group of students is reviewing information about pain transmission and the fibers involved. The students demonstrate understanding when they state that A-delta primary afferent fibers transmit pain that is felt as which of the following? A) Burning B) Throbbing C) Sharp D) Aching

C

2. A new nursing graduate recently made an oversight during the analysis of a client's assessment data that resulted in a postoperative complication. What characteristic of data analysis makes it a challenging aspect of nursing practice? A) Abnormal data must be identified. B) It requires the prior identification of nursing diagnoses. C) It requires sophisticated diagnostic reasoning skills. D) Conclusions must be clearly and accurately documented.

C

2. When assessing the mental status of a 67-year-old woman, the nurse detects some difficulty with free-flow of thought and the woman's ability to follow directions. Which of the following would the nurse do first? A) Use a Geriatric Depression Scale. B) Refer for further medical evaluation. C) Assess the client's vision and hearing. D) Refer the client to social services for home assistance.

C

20. A nurse on the hospital's subacute medical unit is planning to perform a client's focused assessment. Which of the following statements should inform the nurse's practice? A) The focused assessment should be done before the physical exam. B) The focused assessment replaces the comprehensive database. C) The focused assessment addresses a particular client problem. D) The focused assessment is done after gathering subjective data.

C

20. After teaching a group of students about the second phase of the nursing process, the instructor determines that additional teaching is needed when the students identify which of the following as a component? A) Organizing data B) Clustering data C) Formulating a medical diagnosis D) Generating hypotheses

C

21. A hospital's protocols for assessment have been modified in light of standards established by the Joint Commission. What change would bring practice into alignment with these standards? A) Teaching all new clients about the basic pathophysiology of pain B) Assessing clients' pain objectively rather than subjectively C) Identifying pain as the fifth vital sign and assessing clients accordingly D) Triaging clients according to the type of pain that they are experiencing

C

21. An experienced medical-surgical nurse has identified critical thinking as an integral component of diagnostic reasoning. How can the relationship between these two concepts be best described? A) Critical thinking is the practical application of diagnostic reasoning skills. B) Critical thinking and diagnostic reasoning are synonymous. C) Critical thinking is the foundation of the process of diagnostic reasoning. D) Critical thinking is the domain of the novice nurse, whereas diagnostic reasoning is present in experts.

C

21. The nurse is interviewing an 82-year-old client who is accompanied by her daughter. The daughter states that her mother is ìunable to hold her urine,î and the client attests that this is true. What question should the nurse prioritize when assessing the client's urinary incontinence? A) ìDid you deliver your children vaginally or by cesarean section?î B) ìHave you been prone to urinary tract infections in the past?î C) ìIs this something that has begun to happen just recently?î D) ìHave you noticed any change in your bowel function?î

C

21. When applying Kohlberg's theory of moral development to the status of an older adult client, on what assessment finding would the nurse focus? A) The relationship between the client's stated beliefs and his actions B) The client's ability to discern the motivations of others C) The client's adherence to rules, laws, and norms D) The client's ability to tolerate differing views

C

22. An 84-year-old man has been admitted to the emergency department from an extended care facility. Facility staff suspect that the client has pneumonia, and his malaise, productive cough, shortness of breath, and adventitious breath sounds are consistent with this diagnosis. However, the nurse's assessment of the client's vital signs yields an oral temperature of 97.5∞F. How should the nurse best interpret this assessment finding? A) The client likely has a cardiac health problem, not a respiratory health problem. B) The client's signs and symptoms are related to hypothermia rather than infection. C) The client's normothermic temperature does not rule out the presence of an infection. D) The client's infection is no longer localized and has become systemic.

C

22. During an educational inservice, nursing have been encouraged to conduct a self-appraisal of their critical thinking skills. Which of the following questions can best guide this appraisal? A) "Do I tend to make errors in my nursing practice?" B) "Do I get good feedback from clients and their families?" C) "Am I open to the fact that I may not be right?" D) "Am I a resource to my colleagues during a crisis?"

C

22. The nurse is using the mnemonic "COLDSPA" to assess a client's complaint of lower abdominal pain. The nurse asks the client to rate the pain on a scale of 0 to 10. The nurse is assessing which aspect of the complaint? A) Character B) Onset C) Severity D) Pattern

C

23. A woman has accompanied her 80-year-old husband to a scheduled clinic visit and expresses concern about subtle declines in his cognition. Which of the following principles should guide the nurse's assessment of the client's mental status? A) The nurse must modify the cognitive assessment to exclude assessments requiring reading or writing. B) The nurse should first explain to the couple that senility is expected among adults over age 80. C) The nurse must differentiate between age-related changes and the signs and symptoms of dementia. D) The nurse must explain that the results of the assessment will be used to determine if admission to long-term care is necessary.

C

23. An audit of a hospital unit's incident reports reveals that several errors have resulted from incomplete or inaccurate information during change-of-shift handoff. In order to prevent such errors, what practice should be encouraged on the unit? A) Delegate handoff reports to unlicensed care providers who have fewer demands on their time. B) Use an intermediary to receive report from the first nurse and then provide the handoff report to the second nurse. C) Involve as few people as possible in the verbal report. D) Encourage nurses to perform handoff as quickly as possible.

C

23. The nurse is obtaining information about a client's past health history. Which client statement would best reflect this component of assessment? A) "My mom's still alive, but my dad died 10 years ago of heart failure." B) "I have a brother with leukemia and a sister with hypertension." C) "I had surgery 5 years ago to repair an inguinal hernia." D) "I have been having some pain when I urinate for the last several days."

C

24. A female client with bone cancer is experiencing pain that has become more severe over the past several days. When modifying the client's plan of care, the nurse identifies a need to assess the affective dimension of the client's pain. How can the nurse best accomplish this goal? A) Document the ways that the client's pain affects her activities of daily living. B) Determine whether the client is able to independently treat her pain. C) Closely monitor the effects of the client's pain on her emotions. D) Ask the client to rate her pain during every physiological assessment.

C

24. A nurse is teaching a recent nursing graduate about the significance of verbal and nonverbal communication during client care. The new graduate demonstrates an understanding of these techniques by citing what example of verbal communication? A) Maintaining an open attitude B) Using silence appropriately C) Providing a laundry list of descriptors when needed D) Maintaining an open and encouraging facial expression

C

24. The intensive care nurse is working with a client who has increased intracranial pressure secondary to a traumatic brain injury. The nurse is performing the hourly assessment of the client's level of consciousness and observes that the client's eyes are closed. How should the nurse first stimulate the client to assess for arousability? A) Gently shake the client's right shoulder and then his left shoulder. B) Rub the client's sternum with the knuckles. C) Speak to the client clearly from a close distance. D) Press down on one of the client's nail beds.

C

25. A client comes to the health care provider's office for a visit. The client has been seen in this office on occasion for the past 5 years and arrives today complaining of a fever and sore throat. Which type of assessment would the nurse most likely perform? A) Comprehensive assessment B) Ongoing assessment C) Focused assessment D) Emergency assessment

C

25. A hospital nurse is admitting a client with a documented history of acute pancreatitis, liver cirrhosis, malnutrition, and frequent traumatic injuries. What assessment finding would most clearly warrant validation? A) The client's blood pressure is 148/88 mm Hg. B) The client is oriented to person and place but not to time. C) The client states that she only drinks alcohol on a social basis. D) The client states, ìMy skin's kind of yellow because of my liver.î

C

25. The nurse is using her fingerpads to palpate a client's body part during the physical examination. Which of the following would the nurse best be able to detect? A) Temperature B) Vibrations C) Pulses D) Fremitus

C

26. An older adult client who enjoys good overall health has sought care because of a recent onset of weakness and fatigue. The client is unaware of any precipitating events. How should the nurse proceed with assessment? A) Perform a focused respiratory assessment. B) Obtain the client's vaccination history. C) Assess the client for signs and symptoms of anemia. D) Assess the client for evidence of chronic heart failure.

C

28. The nurse has observed that a client adheres rigidly to the norms of her family and her culture. In the context of Freud's theory of development, this pattern of behavior is attributable to the action of what component of personality? A) The id B) The ego C) The superego D) The identity

C

28. The nurse palpates a client's pulse and notes that the rate is 61 beats per minute, with an amplitude that is weak and thready. How should the nurse respond to this assessment finding? A) Call a code blue from the bedside and prepare for resuscitation. B) Assess the client's jugular venous pressure. C) Assess the client's pulse at the carotid site. D) Palpate the client's femoral pulse.

C

28. There has been some resistance to the planned transition to electronic health records (EHRs) in a hospital system, with many caregivers questioning the rationale for this change in practice. What potential advantage of EHRs should administrators cite? A) Increased influence for the nursing profession B) Elimination of documentation C) Improved continuity of care D) Reduced nursing workload

C

29. A nurse is performing a detailed pain assessment of a client who has sought care for debilitating migraines. When assessing for precipitating factors, what question should the nurse ask? A) ìIs there anything that's given you relief in the past?î B) ìHave your migraines gotten more severe in the last few months?î C) ìWhat were you doing immediately before your last migraine?î D) ìHow long does a typical migraine last?î

C

29. A nurse who provides care in a hospital setting is creating a plan of nursing care for a client who has a diagnosis of chronic renal failure. The nurse's plan specifies frequent ongoing assessments. The frequency of these nursing assessments should be primarily determined by what variable? A) The client's age B) The unit's protocols C) The client's acuity D) The nurse's potential for liability

C

29. During the mental status assessment of a new client, the nurse has asked the client to describe some of the similarities and differences between a tennis ball and a soccer ball. Despite adequate time and cuing, the client is unable to state any similarities or differences. The nurse should document what assessment finding? A) A deficit in practical intelligence B) An inability to follow directions accurately C) A deficit in abstract reasoning D) A lack of spatial orientation

C

29. While assisting an older adult with morning hygiene, the nurse notes a lesion on the client's coccyx region. How should the nurse best document this objective assessment finding? A) ìPossible pressure ulcer observed over client's coccyx region.î B) ìReddened area noted on skin surface superficial to client's coccyx.î C) ìArea of nonblanching erythema noted over client's coccyx, 2 cm ◊ 2 cm.î D) ìImpaired Skin Integrity related to decreased mobility.î

C

3. A nurse is admitting a new client to the subacute medical unit and is completing a comprehensive assessment. The nurse is appropriately applying standard precautions by performing which of the following actions? A) Performing hand hygiene between examinations of each body part B) Discarding in the trash can the safety pin that was used to assess sensory perception C) Wearing gloves to palpate the tongue and buccal membranes D) Wearing a gown, gloves, and mask during the physical exam

C

3. A nurse is assessing the pain of a client who has had major surgery. The client also has been experiencing depression. Which of the following principles should guide the nurse's assessment of a client's pain? A) The client is likely experiencing less pain than he is reporting. B) The client's depression exists independently of the level of pain. C) It is likely that the client's pain rating will be influences by his emotional state. D) The degree of surgery will be the key indicator for level of pain experienced.

C

30. A young man has presented to the clinic with a 2-week history of head congestion, fever, and malaise. What assessment technique should the nurse utilize to assess for sinus tenderness? A) Light palpation B) Deep palpation C) Direct percussion D) Blunt percussion

C

5. A nurse has gathered the necessary equipment for the physical assessment of an adult client. For which of the following assessments would it be most appropriate for a nurse to use a centimeter-scale ruler for measurement? A) Mid-arm circumference B) Client's height C) Skin lesion size D) Pupillary size

C

5. A nurse is providing feedback to a colleague after observing the colleague's interview of a newly admitted client. Which of the following would the nurse identify as an example of a closed-ended question or statement? A) "Tell me about your relationship with your children?" B) "Tell me what you eat in a normal day?" C) "Are you allergic to any medications?" D) "What is your typical day like?"

C

5. Which of the following client situations would the nurse interpret as requiring an emergency assessment? A) A pediatric client with severe sunburn B) A client needing an employment physical C) A client who overdosed on acetaminophen D) A distraught client who wants a pregnancy test

C

6. A nurse is admitting a client to the postsurgical unit following breast reconstruction surgery. Which of the following would the nurse use as the primary assessment for the client's pain? A) The client's spiritual view of the pain B) Current pain therapies used preoperatively C) The client's report of her pain D) Psychosocial questions related to her perceptions of pain

C

6. A nurse is preparing a health education class for a group of older adult clients at a local senior center. The nurse is focusing on health promotion and disease prevention. Which condition would the nurse cite as a common cause of infection-related deaths in the elderly? A) Pyelonephritis B) Cellulitis C) Pneumonia D) Meningitis

C

6. A nurse obtains the blood pressure of a client who is uncharacteristically fatigued and who is lying in bed rather than sitting in a chair. The nurse should interpret the client's blood pressure reading in light of what principle? A) The client's blood pressure will be slightly highly than the client's norm. B) Position rarely affects the client's blood pressure. C) The client's blood pressure will be slightly lower than standing readings. D) There will be questionable accuracy of the blood pressure reading.

C

6. A nurse who provides care in a busy, inner-city clinic performs physical examinations on clients of various cultures. In a client from which group would the nurse expect to find the greatest amount of body odor from perspiration? A) Inuit B) Asian C) Caucasian D) Native American

C

6. The nurse is caring for a client with influenza symptoms and is documenting the initial and ongoing assessment database. Which of the following would the nurse emphasize as the major rationale for this action? A) Reducing the fragmentation of care B) Maximizing the efficiency of care C) Promoting communication between disciplines D) Facilitating achievement of professional standards

C

8. The nurse collects vital signs on a hospital client who has recently been experiencing pain. Which of the following would suggest most strongly to the nurse that the client is experiencing pain? A) Respiratory rate of 18 breaths per minute B) Temperature of 99.1∞F C) Heart rate of 110 beats per minute D) Blood pressure of 120/70 mm Hg

C

9. A nurse has assessed a client and identified data that are associated with the diagnoses of Impaired Physical Mobility and Activity Intolerance. How can the nurse best determine which nursing diagnosis is most applicable to the client? A) Document preliminary conclusions. B) Identify abnormal data. C) Check the defining characteristics of the diagnoses. D) Test the nursing diagnoses clinically.

C

9. The nurse is conducting a health interview and is addressing the client's current stressors. What is the primary rationale for including stress as a focus of psychosocial assessment? A) Stress provides the main impetus for psychosocial development and adaptation. B) Psychosocial development cannot progress normally in the presence of stress. C) Psychosocial stress has a major influence on health in many domains. D) The results of the health interview are distorted when the client is experiencing stress.

C

A visiting nurse is making an initial home visit for a patient who has many chronic medical problems. Which type of data base is most appropriate to collect in this setting? a. A follow-up data base to evaluate changes at appropriate intervals b. An episodic data base because of the continuing, complex medical problems of this patient c. A complete health data base because of the nurses primary responsibility for monitoring the patients health d. An emergency data base because of the need to collect information and make accurate diagnoses rapidly

C

The nurse is reviewing information about evidence-based practice (EBP). Which statement best reflects EBP? a. EBP relies on tradition for support of best practices. b. EBP is simply the use of best practice techniques for the treatment of patients. c. EBP emphasizes the use of best evidence with the clinicians experience. d. The patients own preferences are not important with EBP.

C

The nurse recognizes that the concept of prevention in describing health is essential because: a. Disease can be prevented by treating the external environment. b. The majority of deaths among Americans under age 65 years are not preventable. c. Prevention places the emphasis on the link between health and personal behavior. d. The means to prevention is through treatment provided by primary health care practitioners.

C

When considering priority setting of problems, the nurse keeps in mind that second-level priority problems include which of these aspects? a. Low self-esteem b. Lack of knowledge c. Abnormal laboratory values d. Severely abnormal vital signs

C

When listening to a patients breath sounds, the nurse is unsure of a sound that is heard. The nurses next action should be to: a. Immediately notify the patients physician. b. Document the sound exactly as it was heard. c. Validate the data by asking a coworker to listen to the breath sounds. d. Assess again in 20 minutes to note whether the sound is still present.

C

Which of these would be formulated by a nurse using diagnostic reasoning? a. Nursing diagnosis b. Medical diagnosis c. Diagnostic hypothesis d. Diagnostic assessment

C

14. The nurse is completing the assessment of a client who takes a beta-adrenergic blocker and a diuretic. Which assessment would be most important for the nurse to complete to ensure safety with a client receiving antihypertensive agents? A) Noting a widened pulse pressure B) Asking whether the client is experiencing headaches C) Assessing for a rise in blood pressure when standing D) Evaluating for orthostatic hypotension

D

A 42-year-old patient of Asian descent is being seen at the clinic for an initial examination. The nurse knows that including cultural information in his health assessment is important to: a. Identify the cause of his illness. b. Make accurate disease diagnoses. c. Provide cultural health rights for the individual. d. Provide culturally sensitive and appropriate care.

D

10. A 25-year-old type 1 diabetic clerk presents to the emergency room with shortness of breath and states that his blood sugar was 605 at home. You diagnose the patient with diabetic ketoacidosis. What is the expected pattern of breathing? A) Normal B) Rapid and shallow C) Rapid and deep D) Slow

C; Ans: C Chapter: 04 Page and Header: 114, The Vital Signs Feedback: This is the expected rate and depth in diabetic ketoacidosis. The body is trying to rid itself of carbon dioxide to compensate for the acidosis. This is known as Kussmaul's breathing and is seen in other causes of acidosis as well.

In the health promotion model, the focus of the health professional includes: a. Changing the patients perceptions of disease. b. Identifying biomedical model interventions. c. Identifying negative health acts of the consumer. d. Helping the consumer choose a healthier lifestyle.

D

18. The nurse is preparing to assess an adult woman's activities related to health promotion and maintenance. Which question should the nurse ask to obtain the most objective and thorough assessment data? A) "Do you always wear your seatbelt when driving?" B) "How much beer, wine, or alcohol do you drink?" C) "Do you use condoms with each sexual encounter?" D) "Could you describe how you perform self-breast exams?"

D

15. A client has a documented history of hepatomegaly (liver enlargement), and the nurse recognizes the need to perform deep palpation during the physical assessment. The nurse should perform which of the following actions? A) Use one hand and depress the skin 1 centimeter. B) Use the dominant hand to depress the skin one-half to three-quarters of an inch. C) Use both hands to depress the skin one-half of an inch. D) Use both hands to depress the skin 1 to 2 inches.

D

2. A patient comes to the emergency room for evaluation of shortness of breath. To which anatomic region would you assign the symptom? A) Reproductive B) Urinary C) Cardiac D) Hematologic

C; Ans: C Chapter: 02 Page and Header: 27, Assessment and Plan: The Process of Clinical Reasoning Feedback: Cardiac disorders such as congestive heart failure are the most likely on this list to result in shortness of breath. There are cases within the other categories which may also result in shortness of breath, such as anemia in the hematologic category, pregnancy in the reproductive category, or sepsis with UTI in the urinary category. This demonstrates the "tension" in clinical reasoning between making sure all possibilities are covered, while still being able to pick the most likely cause.

22. The nurse has been applying the nursing process in the care of an adult client who is being treated for acute pancreatitis. Place the nurse's actions in their proper sequence from first to last. A) Identifying outcomes B) Determining client's nursing problem C) Collecting information about the client D) Determining outcome achievement E) Carrying out interventions

C, B, A, E, D

25. The admission of a new resident to a long-term care facility has necessitated a thorough health history. Place the following focuses in the correct sequence in which the nurse should perform them, beginning with the section obtained first. A) Family health history B) Reason for seeking care C) Biographic data D) Review of body systems E) History of present concern F) Past health history

C, B, E, F, A, D

1. For which of the following patients would a comprehensive health history be appropriate? A) A new patient with the chief complaint of "I sprained my ankle" B) An established patient with the chief complaint of "I have an upper respiratory infection" C) A new patient with the chief complaint of "I am here to establish care" D) A new patient with the chief complaint of "I cut my hand"

C; Ans: C Chapter: 01 Page and Header: 4, Patient Assessment: Comprehensive or Focused Feedback: This patient is here to establish care, and because she is new to you, a comprehensive health history is appropriate.

6. The following information is recorded in the health history: "The patient completed 8th grade. He currently lives with his wife and two children. He works on old cars on the weekend. He works in a glass factory during the week." Which category does it belong to? A) Chief complaint B) Present illness C) Personal and social history D) Review of systems

C; Ans: C Chapter: 01 Page and Header: 6, The Comprehensive Adult Health History Feedback: Personal and social history information includes educational level, family of origin, current household status, personal interests, employment, religious beliefs, military history, and lifestyle (including diet and exercise habits; use of alcohol, tobacco, and/or drugs; and sexual preferences and history). All of this information is documented in this example.

7. A 19-year old-college student presents to the emergency room with fever, headache, and neck pain/stiffness. She is concerned about the possibility of meningococcal meningitis. Several of her dorm mates have been vaccinated, but she hasn't been. Which of the following physical examination descriptions is most consistent with meningitis? A) Head is normocephalic and atraumatic, fundi with sharp discs, neck supple with full range of motion B) Head is normocephalic and atraumatic, fundi with sharp discs, neck with paraspinous muscle spasm and limited range of motion to the right C) Head is normocephalic and atraumatic, fundi with blurred disc margins, neck tender to palpation, unable to perform range of motion D) Head is normocephalic and atraumatic, fundi with blurred disc margins, neck supple with full range of motion

C; Ans: C Chapter: 02 Page and Header: 27, Assessment and Plan: The Process of Clinical Reasoning Feedback: Blurred disc margins are consistent with papilledema, and neck tenderness and lack of range of motion are consistent with neck stiffness, which in this scenario is likely to be caused by meningeal inflammation. Later, you will learn about Kernig's and Brudzinski's signs, which are helpful in testing for meningeal irritation on examination.

15. A nurse is determining whether the data for a client support a potential nursing diagnosis. The nurse is most likely engaged in which step in the diagnostic reasoning process? A) Step Three: Draw Inferences B) Step Four: Propose Possible Nursing Diagnoses C) Step Five: Check for Defining Characteristics D) Step Six: Confirm or Rule Out Diagnoses

D

8. A 37-year-old nurse comes for evaluation of colicky right upper quadrant abdominal pain. The pain is associated with nausea and vomiting and occurs 1 to 2 hours after eating greasy foods. Which one of the following physical examination descriptions would be most consistent with the diagnosis of cholecystitis? A) Abdomen is soft, nontender, and nondistended, without hepatosplenomegaly or masses. B) Abdomen is soft and tender to palpation in the right lower quadrant, without rebound or guarding. C) Abdomen is soft and tender to palpation in the right upper quadrant with inspiration, to the point of stopping inspiration, and there is no rebound or guarding. D) Abdomen is soft and tender to palpation in the mid-epigastric area, without rebound or guarding.

C; Ans: C Chapter: 02 Page and Header: 27, Assessment and Plan: The Process of Clinical Reasoning Feedback: In cholecystitis, the pain, which originates from the gallbladder, is located in the right upper quadrant. Severity of pain with inspiration that is sufficient to stop further inhalation is also known as Murphy's sign, which, if present, is further indicative of inflammation of the gallbladder.

3. A patient presents for evaluation of a cough. Which of the following anatomic regions can be responsible for a cough? A) Ophthalmologic B) Auditory C) Cardiac D) Endocrine

C; Ans: C Chapter: 02 Page and Header: 27, Assessment and Plan: The Process of Clinical Reasoning Feedback: The cardiac system can cause a cough if the patient has congestive heart failure. This results in fluid buildup in the lungs, which in turn can cause a cough that produces pink, frothy sputum. A foreign body in the ear may also cause a cough by stimulating Arnold's branch of the vagus nerve, but this is less likely to be seen clinically than heart failure.

5. A 47-year-old contractor presents for evaluation of neck pain, which has been intermittent for several years. He normally takes over-the-counter medications to ease the pain, but this time they haven't worked as well and he still has discomfort. He recently wallpapered the entire second floor in his house, which caused him great discomfort. The pain resolved with rest. He denies fever, chills, rash, upper respiratory symptoms, trauma, or injury to the neck. Based on this description, what is the most likely pathologic process? A) Infectious B) Neoplastic C) Degenerative D) Traumatic

C; Ans: C Chapter: 02 Page and Header: 27, Assessment and Plan: The Process of Clinical Reasoning Feedback: The description is most consistent with degenerative arthritis in the neck. The patient has had intermittent symptoms and the questions asked to elicit pertinent negative and positive findings are negative for infectious, traumatic, or neoplastic disease.

6. A 15-year-old high school sophomore comes to the clinic for evaluation of a 3-week history of sneezing; itchy, watery eyes; clear nasal discharge; ear pain; and nonproductive cough. Which is the most likely pathologic process? A) Infection B) Inflammation C) Allergic D) Vascular

C; Ans: C Chapter: 02 Page and Header: 27, Assessment and Plan: The Process of Clinical Reasoning Feedback: This description is most consistent with allergic rhinitis.

14. Mr. Larson is a 42-year-old widowed father of two children, ages 4 and 11. He works in a sales office to support his family. Recently he has injured his back and you are thinking he would benefit from physical therapy, three times a week, for an hour per session. What would be your next step? A) Write the physical therapy prescription. B) Have your office staff explain directions to the physical therapy center. C) Discuss the plan with Mr. Larson. D) Tell Mr. Larson that he will be going to physical therapy three times a week.

C; Ans: C Chapter: 02 Page and Header: 30, Develop a Plan Agreeable to the Patient Feedback: You should discuss your proposed plan with the patient before implementing it. In this case, you and Mr. Larson will need to weigh the benefit of physical therapy against the ability to provide for his family. You may need to consider other ways of helping the patient, perhaps through prescribed back exercises he can do at home. It is a common mistake to implement a plan without coming to an agreement with the patient first.

16. You are excited about a positive test finding you have just noticed on physical examination of your patient. You go on to do more examination, laboratory work, and diagnostic tests, only to find that there is no sign of the disease you thought would correlate with the finding. This same experience happens several times. What should you conclude? A) Consider not doing this test routinely. B) Use this test when you have a higher suspicion for a certain correlating condition. C) Continue using the test, perhaps doing less laboratory work and diagnostics. D) Omit this test from future examinations.

C; Ans: C Chapter: 02 Page and Header: 38, The Challenges of Clinical Data Feedback: This is an example of a sensitive physical finding that lacks specificity. This does not make this a useless test, because the purpose of a screening physical is to find disease. This finding made you consider the associated condition as one of your hypotheses, and this in itself has value. Other possibilities are that you may be doing the maneuver incorrectly or using it on the wrong population. It is important to ask for hands-on help from your instructor when you have a question about a maneuver. Make sure that your information about the maneuver comes from a reliable source as well. All of this information also applies to history questions.

14. You arrive at the bedside of an elderly woman who has had a stroke, affecting her entire right side. She cannot speak (aphasia). You are supposed to examine her. You notice that the last examiner left her socks at the bottom of the bed, and although sensitive areas are covered by a sheet, the blanket is heaped by her feet at the bottom of the bed. What would you do next? A) Carry out your examination, focusing on the neurologic portion, and then cover her properly. B) Carry out your examination and let the nurse assigned to her "put her back together." C) Put her socks back on and cover her completely before beginning the evaluation. D) Apologize for the last examiner but let the next examiner dress and cover her.

C; Ans: C Chapter: 03 Page and Header: 58, Getting Ready: The Approach to the Interview Feedback: It is crucial to make an effort to make a patient comfortable. In this scenario, the patient can neither speak nor move well. Take a moment to imagine yourself in her situation. As a matter of respect as well as comfort, you should cover the patient appropriately and consider returning a little later to do your examination if you feel she is cold. While it is her nurse's job to keep her comfortable, it is also your responsibility, and you should do what you can. It is unacceptable to leave the patient in the same state in which you found her.

8. A nurse is caring for a client who has been admitted with an infected venous ulcer. The nurse determines that the client will need medical interventions as well as nursing interventions. The nurse would identify which of the following? A) Actual nursing diagnosis B) Referral C) Risk nursing diagnosis D) Collaborative problem

D

2. Suzanne, a 25 year old, comes to your clinic to establish care. You are the student preparing to go into the examination room to interview her. Which of the following is the most logical sequence for the patient-provider interview? A) Establish the agenda, negotiate a plan, establish rapport, and invite the patient's story. B) Invite the patient's story, negotiate a plan, establish the agenda, and establish rapport. C) Greet the patient, establish rapport, invite the patient's story, establish the agenda, expand and clarify the patient's story, and negotiate a plan. D) Negotiate a plan, establish an agenda, invite the patient's story, and establish rapport.

C; Ans: C Chapter: 03 Page and Header: 60, Learning About the Patient: The Sequence of the Interview Feedback: This is the most productive sequence for the interview. Greeting patients and establishing rapport allows them to feel more comfortable before "inviting" them to relate their story. After hearing the patient's story, together you establish the agenda regarding the most important items to expand upon. At the end, together you negotiate the plan of diagnosis and treatment.

6. Mr. W. is a 51-year-old auto mechanic who comes to the emergency room wanting to be checked out for the symptom of chest pain. As you listen to him describe his symptom in more detail, you say "Go on," and later, "Mm-hmmm." This is an example of which of the following skilled interviewing techniques? A) Echoing B) Nonverbal communication C) Facilitation D) Empathic response

C; Ans: C Chapter: 03 Page and Header: 68, Building a Therapeutic Relationship: The Techniques of Skilled Interviewing Feedback: This is an example of facilitation. Facilitation can be posture, actions, or words that encourage the patient to say more.

7. Mrs. R. is a 92-year-old retired teacher who comes to your clinic accompanied by her daughter. You ask Mrs. R. why she came to your clinic today. She looks at her daughter and doesn't say anything in response to your question. This is an example of which type of challenging patient? A) Talkative patient B) Angry patient C) Silent patient D) Hearing-impaired patient

C; Ans: C Chapter: 03 Page and Header: 75, Adapting Your Interview to Specific Situations Feedback: This is one example of a silent patient. There are many possibilities for this patient's silence: depression, dementia, the manner in which you asked the question, and so on.

12. On a very busy day in the office, Mrs. Donelan, who is 81 years old, comes for her usual visit for her blood pressure. She is on a low-dose diuretic chronically and denies any side effects. Her blood pressure is 118/78 today, which is well-controlled. As you are writing her script, she mentions that it is hard not having her husband Bill around anymore. What would you do next? A) Hand her the script and make sure she has a 3-month follow-up appointment. B) Make sure she understands the script. C) Ask why Bill is not there. D) Explain that you will have more time at the next visit to discuss this.

C; Ans: C Chapter: 03 Page and Header: 81, Sensitive Topics That Call for Specific Approaches Feedback: Sometimes, the patient's greatest need is for support and empathy. It would be inappropriate to ignore this comment today. She may have relied heavily upon Bill for care and may be in danger. She may be depressed and even suicidal, but you will not know unless you discuss this with her. Most importantly, you should empathize with her by saying something like "It must be very difficult not to have him at home" and allow a pause for her to answer. You may also ask "What did you rely on him to do for you?" Only a life-threatening crisis with another patient should take you out of her room at this point, and you may need to adjust your office schedule to allow adequate time for her today.

11. Mrs. Lenzo weighs herself every day with a very accurate balance-type scale. She has noticed that over the past 2 days she has gained 4 pounds. How would you best explain this? A) Attribute this to some overeating at the holidays. B) Attribute this to wearing different clothing. C) Attribute this to body fluid. D) Attribute this to instrument inaccuracy.

C; Ans: C Chapter: 04 Page and Header: 102, The Health History Feedback: This amount of weight over a short period should make one think of body fluid changes. You may consider a kidney problem or heart failure in your differential. The other reasons should be considered as well, but this amount of weight gain over a short period usually indicates causes other than excessive caloric intake. A rule of thumb for dieters is that an energy excess of 3500 calories will cause a 1-pound weight gain, if the increase is to be attributed to food intake.

5. Common or concerning symptoms to inquire about in the General Survey and vital signs include all of the following except: A) Changes in weight B) Fatigue and weakness C) Cough D) Fever and chills

C; Ans: C Chapter: 04 Page and Header: 102, The Health History Feedback: This symptom is more appropriate to the respiratory review of systems.

2. A 25-year-old radio announcer comes to the clinic for an annual examination. His BMI is 26.0 kg/m2 . He is concerned about his weight. Based on this information, what is appropriate counsel for the patient during the visit? A) Refer the patient to a nutritionist because he is anorexic. B) Reassure the patient that he has a normal body weight. C) Give the patient information about reduction of fat, cholesterol, and calories because he is overweight. D) Give the patient information about reduction of fat and cholesterol because he is obese.

C; Ans: C Chapter: 04 Page and Header: 104, Health Promotion and Counseling Feedback: The patient has a BMI in the overweight range, which is 25.0 to 29.9 kg/m2 . It is prudent to give him information about reducing calories, fat, and cholesterol in his diet to help prevent further weight gain.

15. Despite having high BP readings in the office, Mr. Kelly tells you that his readings at home are much lower. He checks them twice a day at the same time of day and has kept a log. How do you respond? A) You diagnose "white coat hypertension." B) You assume he is quite nervous when he comes to your office. C) You question the accuracy of his measurements. D) You question the accuracy of your measurements.

C; Ans: C Chapter: 04 Page and Header: 114, The Vital Signs Feedback: It is not uncommon to see differences in a patient's home measurements and your own in the office. Presuming that this is "white coat hypertension" can be dangerous because this condition is not usually treated. This allows for the effects of a missed diagnosis of hypertension to go unchecked. It is also very difficult to judge if a patient is outwardly nervous. You should always consider that your measurements are not accurate as well, but the fact that you and your staff are well-trained and perform this procedure on hundreds of patients a week makes this less likely. Ideally, you would ask the patient to bring in his BP equipment and take a simultaneous reading with you to make sure that he is getting an accurate reading.

19. Ms. Wright comes to your office, complaining of palpitations. While checking her pulse you notice an irregular rhythm. When you listen to her heart, every fourth beat sounds different. It sounds like a triplet rather than the usual "lub dup." How would you document your examination? A) Regular rate and rhythm B) Irregularly irregular rhythm C) Regularly irregular rhythm D) Bradycardia

C; Ans: C Chapter: 04 Page and Header: 119, Heart Rate and Rhythm Feedback: Because this unusual beat occurs every fourth set of heart sounds, it is regularly irregular. This is most consistent with ventricular premature contractions (or VPCs). This is generally a common and benign rhythm. An irregularly irregular rhythm is a classic finding in atrial fibrillation. The rhythm is very random in character. Bradycardia refers to the rate, not the rhythm.

13. A young woman comes to you with a cut on her finger caused by the lid of a can she was opening. She is pacing about the room, crying loudly, and through her sobs she says, "My career as a pianist is finished!" Which personality type exhibits these features? A) Narcissistic B) Paranoid C) Histrionic D) Avoidant

C; Ans: C Chapter: 05 Page and Header: 136, Symptoms and Behavior Feedback: The theatrical nature of her behavior as well as her overreaction lead to a diagnosis of histrionic character disorder.

4. A 32-year-old white female comes to your clinic, complaining of overwhelming sadness. She says for the past 2 months she has had crying episodes, difficulty sleeping, and problems with overeating. She says she used to go out with her friends from work but now she just wants to go home and be by herself. She also thinks that her work productivity has been dropping because she just is too tired to care or concentrate. She denies any feelings of guilt or any suicidal ideation. She states that she has never felt this way in the past. She denies any recent illness or injuries. Her past medical history consists of an appendectomy when she was a teenager; otherwise, she has been healthy. She is single and works as a clerk in a medical office. She denies tobacco, alcohol, or illegal drug use. Her mother has high blood pressure and her father has had a history of mental illness. On examination you see a woman appearing her stated age who seems quite sad. Her facial expression does not change while you talk to her and she makes little eye contact. She speaks so softly you cannot always understand her. Her thought processes and content seem unremarkable. What type of mood disorder do you think she has? A) Dysthymic disorder B) Manic (bipolar) disorder C) Major depressive episode

C; Ans: C Chapter: 05 Page and Header: 160, Table 5-2 Feedback: Major depression occurs in a person with a previously normal state of mood. The symptoms often consist of a combination of sadness, decreased interest, sleeping problems (insomnia or hypersomnia), eating problems (decreased or increased appetite), feelings of guilt, decreased energy, decreased concentration, psychomotor changes (retardation or agitation), and a preoccupation with thoughts of death or suicide. There must be at least five symptoms for a diagnosis of major depression. This patient has six: (1) sadness, (2) trouble sleeping, (3) overeating, (4) fatigue, (5) difficulty with concentration, and (6) no interest in doing things.

2. A 24-year-old secretary comes to your clinic, complaining of difficulty sleeping, severe nightmares, and irritability. She states it all began 6 months ago when she went to a fast food restaurant at midnight. While she was waiting in her car a man entered through the passenger door and put a gun to her head. He had her drive to a remote area, where he took her money and threatened to kill her. When the gun jammed he panicked and ran off. Ever since this occurred the patient has been having these symptoms. She states she jumps at every noise and refuses to drive at night. She states her anxiety has had such a marked influence on her job performance she is afraid she will be fired. She denies any recent illnesses or injuries. Her past medical history is unremarkable. On examination you find a nervous woman appearing her stated age. Her physical examination is unremarkable. You recommend medication and counseling. What anxiety disorder to you think this young woman has? A) Specific phobia B) Acute stress disorder C) Post-traumatic stress disorder D) Generalized anxiety disorder

C; Ans: C Chapter: 05 Page and Header: 161, Table 5-3 Feedback: Post-traumatic stress disorder is the fearful response (nightmares, avoidance of areas, irritability) to an event that occurred at least 1 month prior to presentation. The patient's fears and reactions cause marked distress and impair social and occupational functions.

7. A 26-year-old violinist comes to your clinic, complaining of anxiety. He is a first chair violinist in the local symphony orchestra and has started having symptoms during performances, such as sweating, shaking, and hyperventilating. It has gotten so bad that he has thought about giving up his first chair status so he does not have to play the solo during one of the movements. He says that he never has these symptoms during rehearsals or when he is practicing. He denies having any of these symptoms at any other time. His past medical history is unremarkable. He denies any tobacco use, drug use, or alcohol abuse. His parents are both healthy. On examination you see a young man who appears worried. His vital signs and physical examination are unremarkable. What type of anxiety disorder best describes his situation? A) Panic disorder B) Specific phobia C) Social phobia D) Generalized anxiety disorder

C; Ans: C Chapter: 05 Page and Header: 161, Table 5-3 Feedback: Social phobia is a marked, persistent fear of social or performance situations.

15. Blood pressure abnormalities found more commonly in Western elderly include which of the following? A) Isolated elevation of the diastolic BP B) Narrow pulse pressure C) Elevation of the systolic BP D) Elevation of the BP with standing

C; Ans: C Chapter: 20 Page and Header: 895, Anatomy and Physiology Feedback: Isolated systolic hypertension is common in the elderly because of stiffening of the large arteries. This is often accompanied by widening of the pulse pressure. Orthostatic BP changes are often seen with postural changes and can account for falls as well.

17. Mrs. Stanton is a 79-year-old widow who presents to your office for a routine BP visit. You note a new pulsatile mass in the right neck at the carotid artery. Which of the following is the most likely cause for this? A) Anxiety B) Carotid artery aneurysm C) Kinking of the artery D) Tortuous aorta

C; Ans: C Chapter: 20 Page and Header: 897, Anatomy and Physiology Feedback: While a carotid artery aneurysm is a possibility, it is more likely due to kinking of the carotid artery in this patient with HTN. A tortuous aorta will sometimes cause elevation of the left jugular vein by impairing drainage within the thorax.

3. A 79-year-old retired banker comes to your office for evaluation of difficulty with urination; he gets up five to six times per night to urinate and has to go at least that often in the daytime. He does not feel as if his bladder empties completely; the strength of the urinary stream is diminished. He denies dysuria or hematuria. This problem has been present for several years but has worsened over the last 8 months. You palpate his prostate. What is your expected physical examination finding, based on this description? A) Normal size, smooth B) Normal size, boggy C) Enlarged size, smooth D) Enlarged size, boggy

C; Ans: C Chapter: 20 Page and Header: 899, Anatomy and Physiology Feedback: This is the expected physical examination finding in benign prostatic hyperplasia (BPH).

22. You are examining an elderly man and notice the following: decreased vibration sense in the feet and ankles, diminished gag reflex, right patellar reflex less than the left, and diminished abdominal reflexes. Which of these is abnormal? A) Decreased vibration sense B) Diminished gag reflex C) Diminished right patellar reflex compared to the left D) Diminished abdominal reflexes

C; Ans: C Chapter: 20 Page and Header: 901, Anatomy and Physiology Feedback: Asymmetry on any examination is usually reason for concern. The other changes are commonly associated with aging. You may consider looking for other neurologic signs on the right, although occasionally you may mistake an abnormally brisk reflex to be normal when compared to the other side. It is usually a good idea to question whether the opposite side is actually the abnormal one when you find asymmetry on examination.

7. An 88-year-old retired piano teacher comes for evaluation of fatigue. You notice that her clothes are hanging loosely off her frame and that she has lost 15 pounds. She is unaware of this. Her husband of 63 years died a few months ago. You ask the patient to complete a Rapid Screen for Dietary Intake. Which of the following statements is considered to be part of this rapid screen? A) I eat more than two meals per day. B) I drink one glass of alcohol every day. C) Without wanting to, I have lost or gained 10 pounds in the last 6 months. D) I eat with at least one other person most of the time.

C; Ans: C Chapter: 20 Page and Header: 906, The Health History Feedback: This is part of the Rapid Screen for Dietary Intake.

24. Mr. Kelly comes to you today for a burning pain in his lower abdomen. This has gone on for 2 months. He has received radiation for prostatic cancer for the past quarter. What assumptions could you draw from this? A) This represents persistent pain. B) His pain reporting is likely to be unreliable. C) There are "red flags" present. D) He is depressed.

C; Ans: C Chapter: 20 Page and Header: 906, The Health History Feedback: This scenario is consistent with acute pain, although this may become persistent if it lasts beyond 3 months. The burning quality to the pain should be a red flag, along with descriptions of pain as "discomfort" or "soreness." Depressed affect or changes in posture or gait are also red flags. Studies have found that pain reporting in the elderly is accurate. Although depression may be present, we have no indications of it in this scenario.

27. Mrs. Geller is somewhat quiet today. She has several bruises of different colors on the ulnar aspects of her forearms and on her abdomen. She otherwise has no complaints and her diabetes and hypertension are well managed. Her son from out of state accompanies her today and has recently moved in to help her. What should you suspect? A) Overuse of aspirin B) Frequent falls C) Elder abuse D) Depression

C; Ans: C Chapter: 20 Page and Header: 912, Health Promotion and Counseling Feedback: The different colors of the bruising indicate that they have occurred at different times and are unlikely to have resulted from a single fall. The location of the bruising on the ulnar aspects of the forearms potentially indicates that she was trying to defend herself and are not typical areas to be bruised by a fall. Depression may be evident, but this is more likely to be a result rather than a cause of her situation today. While nothing is proven, it would be wise to interview her without her son in the room. If in doubt, a social worker consult may be helpful to determine if elder abuse is occurring.

28. A patient comes to you for the appearance of red patches on his forearms that have been present for several months. They remain for several weeks. He denies a history of trauma. Which of the following is likely? A) Actinic keratoses B) Pseudoscars C) Actinic purpura D) Cherry angiomas

C; Ans: C Chapter: 20 Page and Header: 917, Techniques of Examination Feedback: Actinic purpura is a common benign skin condition of the elderly, frequently involving the forearms. Pseudoscars are white patches and cherry angiomas are bright-red raised lesions usually found on the torso. Actinic keratoses are lesions resembling nevi, often with features which would be concerning if considering melanoma (review the ABCDEs of melanoma), but they produce a slightly greasy scale when scratched with a nail.

14. It is summer and an 82-year-old woman is brought to you from her home after seeing her primary care doctor 2 days ago. She was started on an antibiotic at that time. Today, she comes to the emergency room not knowing where she is or what year it is. What could be a likely cause of this? A) Alzheimer's dementia B) Stroke C) Delirium D) Meningitis

C; Ans: C Chapter: 20 Page and Header: 931, Table 20-2 Feedback: These are not signs of normal aging and seem to be of acute onset. This makes Alzheimer's less likely. Stroke and meningitis could cause these symptoms as well, but the combination of the heat and a recent infection make delirium much more likely. Though she was prescribed an antibiotic, she may not have improved because of bacterial resistance or because of noncompliance due to cost, depression, or even an underlying mild dementia. Dementia should not result in an acute mental status change, although illness may cause a worsening of dementia.

15. What action on the part of a middle-aged client would best exemplify Erikson's concept of generativity? A) Being able to accurately evaluate the merits of others' ideas B) Emphasizing the importance of one's knowledge and skill set C) Consistently increasing one's income D) Guiding and mentoring individuals who are younger

D

15. When describing the expansion of the depth and scope of nursing assessment over the past several decades, which of the following would the nurse identify as being the primary force? A) Documentation B) Informatics C) Diversification D) Technology

D

16. The emergency department (ED) nurse is assessing for kidney tenderness in a client who has presented with complaints of dysuria and back pain. What assessment technique should the nurse utilize? A) Deep palpation B) Indirect percussion C) Moderate palpation D) Blunt percussion

D

8. The nurse is assessing a client's cultural identity and affiliation during the health interview. How best can the nurse elicit this information? A) ìWhat are your race and culture?î B) ìWould you describe yourself as American?î C) ìHow would you describe your cultural values?î D) ìWith which cultural group do you most closely identify?î

D

9. A nurse is working in a health care facility that uses charting by exception. Which of the following would the nurse expect to document? A) Liver palpation normal B) No tenderness on palpation C) Bowel sounds normoactive D) Decreased range of motion in right shoulder

D

16. The nurse's interview with an older adult client reveals that he bitterly regrets some of the financial decisions that he made when he was younger. The nurse recognizes that unless the client is able to accept these undesirable aspects of life, what outcome is likely? A) The client will adopt antisocial behaviors late in life. B) The client will die prematurely. C) The client will gradually abandon significant relationships. D) The client will live with despair during his final years of life.

D

17. When considering the various cultural aspects associated with death rituals, which of the following should guide a nurse's practice? A) Most cultures have similar durations for the length of time a person grieves. B) A person's view of death is likely to be different from the original ethnic group's practice. C) Responses to death and grief are fairly consistent among different cultures. D) Rituals for burial and bereavement are likely to reflect original cultural practices.

D

12. An older adult client has been admitted to the hospital with failure to thrive resulting from complications of diabetes. Which of the following would the nurse implement in response to a collaborative problem? A) Encourage the client to increase oral fluid intake. B) Provide the client with a bedtime protein snack. C) Assist the client with personal hygiene. D) Measure the client's blood glucose four times daily.

D

12. The nurse is assessing a client using the Glasgow Coma Scale following an acute hypoglycemic episode and obtains a score of 14. The nurse interprets this as indicating which of the following? A) Deep coma B) Coma C) Obtunded D) Alert and oriented

D

12. The nurse is assessing an older adult's psychosocial development with reference to Freud's theory of development. What observation by the nurse would most clearly suggest healthy development within this theoretical framework? A) The client is able to describe challenges that he has overcome. B) The client has eliminated conflictual relationships from his life. C) The client is able to delegate care to others when necessary. D) The client appears to have dealt effectively with recent losses.

D

13. The nurse is preparing to lead a health promotion activity among a group of clients from different cultures. The nurse would expect that which client would require the least amount of personal space? A) Latin American B) Asian C) American D) Middle Easterner

D

13. When assessing an elderly client's hip joint after a fall, which of the following should lead the nurse to suspect that the client has a hip fracture? A) Internal rotation of the affected leg B) Abduction of the affected leg C) Partial weight bearing D) Thigh pain

D

1. A client has presented to the clinic for the treatment of an ovarian cyst. Which of the following would be most important for the nurse to do immediately before performing this woman's physical exam? A) Explain the purpose of the interview to the client. B) Construct the client's family genogram. C) Establish the client's reliability as historian. D) Collect necessary equipment essential to the exam.

D

1. A nurse on a postsurgical unit is admitting a client following the client's cholecystectomy (gall bladder removal). What is the overall purpose of assessment for this client? A) Collecting accurate data B) Assisting the primary care provider C) Validating previous data D) Making clinical judgments

D

1. The nurse is conducting a functional assessment of an older adult client. The nurse should focus questions on which area? A) Feelings about aging B) Quality of life C) Recent personal losses D) Activities of daily living

D

10. A nurse is preparing to document a collaborative problem for a client with pain. Which of the following would be most appropriate? A) ìImpaired physical mobility related to chronic painî B) ìRisk for powerlessness related to chronic painî C) ìReadiness for enhanced comfort levelî D) ìRC: peripheral nerve compressionî

D

10. The nurse is preparing to perform a physical examination on a female client who has been transferred to the medical unit from the emergency department. The nurse should begin the collection of objective data with which of the following examinations? A) Head and neck examination B) Palpation of lymph nodes C) Breast examination D) Vital signs

D

10. Upon entering an exam room, the client states, "Well! I was getting ready to leave. My schedule is very busy and I don't have time to waste waiting until you have the time to see me!" Which response by the nurse would be most appropriate? A) "Our schedule is very busy also. We got to you as soon as we could." B) "No one is forcing you to be here, and you are free to leave at any time." C) "Would you like to report your complaints to someone with power?" D) "You're certainly justified in being upset, but I am ready to begin your exam now."

D

11. A nurse has admitted a client to the medical unit and is describing the purpose for obtaining a comprehensive health history. Which of the following purposes should the nurse describe? A) "This helps us to complete your health record accurately." B) "This helps us to establish a trusting interpersonal relationship." C) "This helps us to evaluate the seriousness of your risk factors for disease." D) "This helps us have an appropriate focus for the physical examination."

D

11. The nurse is to collect a throat culture from a client who has signs and symptoms of a respiratory infection, including frequent, productive coughing. The nurse demonstrates the best adherence to standard precautions by using which of the following pieces of equipment? A) Eye goggles B) Face mask C) Cover gown D) Face shield

D

11. When assessing an older adult client with osteoporotic thinning and vertebral collapse, which of the following findings would the nurse expect to identify? A) Lordosis B) Increased arm swing C) Narrowed gait D) Kyphosis

D

20. A cardiac care nurse works with a diverse client population. The nurse would assess a client from which cultural group for an increased effect of an antihypertensive medication? A) Eskimos B) Native Americans C) Hispanics D) Chinese

D

20. The nurse has identified abnormal findings when reviewing a young adult client's health history. Within Kohlberg's theory of psychosocial development, what behavioral characteristic is the nurse most likely to observe? A) The client has difficulty trusting others. B) The client is easily manipulated by others. C) The client is unable to weigh options when presented with a dilemma. D) The client makes decisions without considering the impact on others.

D

21. A nurse is completing a general survey of a client's health and is beginning by measuring the client's vital signs. What assessment question constitutes the ìfifth vital signî? A) ìCan you tell me the date and month?î B) ìCan I check your oxygen saturation level?î C) ìAre you experiencing any shortness of breath?î D) ìAre you having any pain right now?î

D

22. A client has presented to the emergency department (ED) with a lower leg laceration that she suffered ìwhile I was on a bender last night.î The nurse recognizes the need to screen for alcohol use and will implement the CAGE questionnaire. What question will the nurse ask during this assessment? A) ìHave you ever experienced a memory blackout after drinking?î B) ìHave you ever vomited blood after drinking alcohol?î C) ìHave you ever been treated for alcohol abuse?î D) ìHave you ever felt guilty about your alcohol use?î

D

22. A nurse is participating in an educational exercise in which she is conducting a self- examination of her own biases. This activity addresses what construct of cultural competence? A) Cultural desire B) Cultural knowledge C) Cultural skill D) Cultural awareness

D

22. During the health interview, a client demonstrates the ability to describe healthy and unhealthy aspects of her thinking patterns. The nurse would conclude that this client has attained which level of development within Piaget's framework? A) Circular operational B) Preoperational C) Concrete operational D) Formal operational

D

23. A nurse is providing care for an 84-year-old client who has diagnoses of middle-stage Alzheimer disease and a femoral head fracture. What assessment tool should the nurse use to assess the client's pain? A) Graphic Rating Scale B) Numeric Rating Scale (NRS) C) Verbal Descriptor Scale D) Faces Pain Scale-Revised (FPS-R)

D

23. Assessment reveals that a young adult has failed to achieve Erikson's central task of his current stage of development. What nursing diagnosis would the nurse associate most closely with this finding? A) Risk for compromised human dignity B) Anxiety C) Ineffective sexuality pattern D) Social isolation

D

23. The nurse is performing an assessment of a hospital client at the beginning of a shift. When assessing the client's heart rate, the nurse will most likely palpate what artery? A) Femoral artery B) Aorta C) Ulnar artery D) Radial artery

D

24. A client has illuminated his call light and tells the nurse that he is having ìten out of tenî pain. The nurse's initial inspection reveals that the client is watching videos on his tablet computer and appears to be at ease physically and emotionally. How should the nurse validate the client's subjective complaint of pain? A) Ask the client to repeat his rating of his pain. B) Observe the client for several seconds to see if his demeanor or his behavior changes. C) Consult the client's medication administration record (MAR) to check for recent analgesic use. D) Perform further assessments addressing various aspects of the client's pain.

D

25. A nurse has been clustering the data that he collected during the initial assessment of a frail elderly client. When making inferences about the data clusters, the nurse is unsure whether to associate a cluster of data with a nursing diagnosis or with a collaborative problem. What question may best guide the nurse's decision? A) "Can an unlicensed care provider meet this person's needs?" B) "Is this problem acute or is it chronic?" C) "Can this issue be addressed on an outpatient basis?" D) "Does this issue require medical intervention?"

D

25. A nurse is conducting a mental status assessment of a 70-year-old male client who is being treated for depression. When assessing the client's facial expression and eye contact, the nurse should consider which of the following? A) The nurse should inform the client that his facial expression is being assessed. B) Reduced eye contact is an age-related physiological change. C) Facial expression should be disregarded if the client has a diagnosed mental illness. D) Eye contact is strongly influenced by cultural norms.

D

25. A nurse is validating assessment findings with a client, and the client proceeds to describe some of the psychological and spiritual components that she believes underlie her disease process. This understanding of the cause of illness is most closely associated with which of the following? A) Northern European cultures B) The Western biomedical model C) African-American culture D) Asian cultures

D

26. A nurse is working with a 22-year-old woman of Asian ethnicity who has been diagnosed with bipolar disorder. When planning culturally appropriate care, the nurse should consider which of the following? A) There may a lack of acceptance that the client's behavior is abnormal. B) The client's family may see her illness as punishment for misdeeds. C) The client's family may see her psychiatric disorder as evidence of bad character. D) There may be shame associated with having a psychiatric disorder.

D

26. A small, rural hospital is revising the policies and procedures surrounding documentation in an effort to align practices with the Health Information Technology for Economic and Clinical Health (HITECH) Act. How can the requirements of this legislation best be met? A) Expand the use of the Nursing Minimum Data Set. B) Eliminate the use of verbal handoffs between nurses. C) Increase interdisciplinary collaboration in the hospital. D) Increase the use of electronic health records (EHRs) in the hospital.

D

26. The nurse has assisted a 74-year-old woman from a chair to the examination table during an assessment, and the nurse observes that the client moves particularly slowly and stiffly. The nurse should question the client regarding a possible history of what health problem? A) Rhabdomyolysis B) Diabetes C) Kyphosis D) Arthritis

D

27. The nurse is completing an assessment of a 50-year-old female client who has sought care for recurrent migraines that have not responded to treatment. Following the review of systems, how should the nurse best document unremarkable results of the subjective portion of the gastrointestinal assessment? A) "Client's gastrointestinal health is within reference ranges for age." B) "Client denies GI signs and symptoms." C) "Gastrointestinal problems are absent." D) "Client denies recent constipation, diarrhea, bowel incontinence, or abdominal pain."

D

27. The nurse's assessment of a client with a decreased level of consciousness reveals that the client is incontinent of urine. During the process of data analysis, the nurse would be justified in identifying what risk nursing diagnosis? A) Risk for Injury related to urinary incontinence B) Risk for Infection related to urinary incontinence C) Risk for Bowel Incontinence related to urinary incontinence D) Risk for Impaired Skin Integrity related to urinary incontinence

D

28. A nurse will complete an initial comprehensive assessment of a 60-year-old client who is new to the clinic. What goal should the nurse identify for this type of assessment? A) Identify the most appropriate forms of medical intervention for the client. B) Determine the most likely prognosis for the client's health problem. C) Identify the status of the client's airway, breathing, and circulation. D) Establish a baseline for the comparison of future health changes.

D

28. An 88-year-old woman has been admitted to the acute medical unit for the treatment of a urinary tract infection that is thought to be progressing to urosepsis. When assessing the client's orientation, how should the nurse best gauge the client's orientation to time? A) ìCan you tell me approximately what time it is right now?î B) ìAre you able to tell me today's date?î C) ìCan you tell me the date and the day of the week?î D) ìAre you able to tell the month and the year that we're in?î

D

28. The nurse is reviewing an older adult's recent laboratory values prior to performing a physical assessment. What value would most clearly indicate the need for further nutritional assessment? A) Hemoglobin 12.2 g/dL B) Hematocrit 40% C) Serum albumin 3.9 g/dL D) Vitamin B12 91 μg/ml

D

29. An older adult client has come to the clinic with new complaints of fatigue, constipation, and cold intolerance. This client may benefit from referral for which of the following purposes? A) Liver function testing B) Cognitive testing C) Lung function testing D) Assessment of thyroid function

D

29. Data analysis of assessment data from a client who presented to the emergency department has resulted in the nurse making a syndrome nursing diagnosis. What is a primary characteristic of this type of diagnosis? A) The client's health problem cannot be conveyed using standard nursing language. B) The client's current signs and symptoms are the result of a longstanding health problem. C) The client has health problems that will require multidisciplinary care. D) The client has a number of nursing diagnoses that typically occur together.

D

3. A hospital nurse has identified a need to improve her critical thinking skills in an effort to improve client care. The nurse should identify which of the following characteristics of critical thinking? A) It is an innate skill that some individuals possess and which others do not. B) It does not include past experiences. C) It is based primarily on getting correct and timely information. D) It involves reflections on thoughts before reaching conclusions.

D

3. A school nurse who provides care in a middle school works exclusively with adolescents. According to Erikson's theory of psychosocial development, what task will underlie much of the students' behavior? A) Exerting influence over others B) Evaluating the merits of their parents' beliefs C) Appraising religious dogma D) Establishing a personal identity

D

30. A nurse is obtaining subjective data from an adult client who is new to the clinic. The nurse has asked the client, "Where do you usually turn for help in a time of crisis?" What domain is this nurse assessing? A) The client's family relationships B) The client's current level of social and relational stability C) The client's critical thinking and problem-solving abilities D) The client's stress management and coping strategies

D

4. When describing the importance of documenting initial assessment data to a group of new nurses, which of the following would the nurse emphasize as the primary reason? A) Health care institutions have established policies regarding documentation. B) Incorrect conclusions may be made without documentation of the nurse's opinions. C) It satisfies legal standards established by health care organizations and institutions. D) It becomes the foundation for the entire nursing process.

D

4. When examining the eyes of an elderly client, the nurse observes a brownish discoloration of the lens. The nurse interprets this finding as being suggestive of what health problem? A) Conjunctivitis B) Presbyopia C) Glaucoma D) Cataracts

D

5. A nurse educator is presenting an in-service program to a group of nurses who will be working on an oncology unit. Which of the following characteristics of cancer pain should the nurse describe? A) Its basis is usually chronic neuropathy. B) It is most often caused by a specific recent trauma. C) It usually appears in the first month after cancer develops. D) It is typically caused by compressed peripheral nerves.

D

5. A nurse is working in a clinic in a low-income neighborhood and assesses a female adult client who states that she has a urinary tract infection. The nurse notes that the client is unkempt, wearing stained clothing, and has a strong body odor. The client mentions that she was evicted from her apartment two weeks ago. Which nursing diagnosis would the nurse most likely identify for this client? A) Caregiver role strain related to fatigue B) Impaired skin integrity related to neurologic deficits C) Deficient fluid volume related to possible urinary tract infection D) Self-care deficit related to possible homelessness

D

5. The nurse is conducting an assessment of an older adult client who has a diagnosis of chronic heart failure. How can the nurse best assess the effects of the client's stroke volume? A) Take the blood pressure while the client is standing. B) Measure the strength of the radial pulse. C) Add the radial pulse and the systolic blood pressure. D) Calculate the difference between the diastolic and systolic pressures.

D

6. A nurse is planning a client's care following the completion of an initial assessment. When formulating a risk nursing diagnosis, which piece of data would be most useful? A) The client has an elevated white blood cell count. B) The client is 66 years of age. C) The client has pain in her joints, especially in the morning. D) The client is separated from her usual social supports.

D

6. When preparing to obtain information about a client's mental and psychosocial status, which of the following would the nurse need to do first? A) Question the patient about his or her usual lifestyle and behaviors. B) Perform a neurologic examination to determine any deficits. C) Check the client's level of consciousness for changes. D) Explain the purpose of the exam and types of questions.

D

7. During the assessment interview, the client made numerous statements that suggested his life generally exists in a state of harmony and balance. This fact would most likely prompt the nurse to identify which of the following? A) Actual nursing diagnosis B) Risk nursing diagnosis C) Collaborative problem D) Health promotion diagnosis

D

7. The nurse is completing an initial assessment of a client who is new to the ambulatory clinic. Before assessing the client's blood pressure, a nurse asks him what his usual blood pressure is. The nurse bases this action primarily on what rationale? A) It provides identifiable data about the client. B) It verifies the client's cardiac function. C) It assesses the client's distant memory recall. D) It indicates the client's involvement in his health care.

D

7. The nurse is using the Verbal Descriptor Scale to assess a client's pain. The nurse will prioritize which of the following data? A) The client's facial expressions B) The client's report on a 0 to 10 numeric scale C) The client's rating on a 0 to 10 visual analog scale D) The client's explanation of how her pain feels

D

8. A client's medical assessment reveals no heart disease. An electrocardiogram is performed and a dysrhythmia is noted. The nurse interprets this finding as most likely reflecting which of the following age-related changes? A) Decreased ventricular compliance B) Peripheral vascular disease C) Widening pulse pressure D) Collagen deposits around pacemaker cells

D

8. A nurse has identified the goal of becoming more culturally sensitive and competent. What is the primary rationale for cultural sensitivity in health care settings? A) Recognize that cultural diversity exists. B) Understand individual differences. C) Prevent offending the client. D) Obtain accurate assessment data.

D

The nurse is conducting a class on priority setting for a group of new graduate nurses. Which is an example of a first-level priority problem? a. Patient with postoperative pain b. Newly diagnosed patient with diabetes who needs diabetic teaching c. Individual with a small laceration on the sole of the foot d. Individual with shortness of breath and respiratory distress

D

The nurse is performing a physical assessment on a newly admitted patient. An example of objective information obtained during the physical assessment includes the: a. Patients history of allergies. b. Patients use of medications at home. c. Last menstrual period 1 month ago. d. 2 5 cm scar on the right lower forearm.

D

The nursing process is a sequential method of problem solving that nurses use and includes which steps? a. Assessment, treatment, planning, evaluation, discharge, and follow-up b. Admission, assessment, diagnosis, treatment, and discharge planning c. Admission, diagnosis, treatment, evaluation, and discharge planning d. Assessment, diagnosis, outcome identification, planning, implementation, and evaluation

D

What step of the nursing process includes data collection by health history, physical examination, and interview? a. Planning b. Diagnosis c. Evaluation d. Assessment

D

When reviewing the concepts of health, the nurse recalls that the components of holistic health include which of these? a. Disease originates from the external environment. b. The individual human is a closed system. c. Nurses are responsible for a patients health state. d.Holistic health views the mind, body, and spirit as interdependent.

D

Which situation is most appropriate during which the nurse performs a focused or problem-centered history? a. Patient is admitted to a long-term care facility. b. Patient has a sudden and severe shortness of breath. c. Patient is admitted to the hospital for surgery the following day. d. Patient in an outpatient clinic has cold and influenza-like symptoms.

D

The nurse makes which adjustment in the physical environment to promote the success of an interview? a. Reduces noise by turning off televisions and radios b. Reduces the distance between the interviewer and the patient to 2 feet or less c. Provides a dim light that makes the room cozy and helps the patient relax d. Arranges seating across a desk or table to allow the patient some personal space

a

11. Steve has just seen a 5-year-old girl who wheezes when exposed to cats. The patient's family history is positive for asthma. You think the child most likely has asthma. What have you just accomplished? A) You have tested your hypothesis. B) You have developed a plan. C) You have established a working diagnosis. D) You have created a hypothesis.

D; Ans: D Chapter: 02 Page and Header: 27, Assessment and Plan: The Process of Clinical Reasoning Feedback: As you go through a history and examination, you will start to generate ideas to explain the patient's symptoms. It is best to keep an open mind and make as many hypotheses as you can, to avoid missing a possibility. A common mistake is to latch onto one idea too early. Once you have committed your mind to a diagnosis, it is difficult to change to another. To think about looking for wheezes on examination would be an example of testing your new hypothesis. Starting a patient on an inhaled medicine would be a plan. It is too early to commit to a working diagnosis, given the amount of information you have gathered.

15. You are seeing an elderly man with multiple complaints. He has chronic arthritis, pain from an old war injury, and headaches. Today he complains of these pains, as well as dull chest pain under his sternum. What would the order of priority be for your problem list? A) Arthritis, war injury pain, headaches, chest pain B) War injury pain, arthritis, headaches, chest pain C) Headaches, arthritis, war injury pain, chest pain D) Chest pain, headaches, arthritis, war injury pain

D; Ans: D Chapter: 02 Page and Header: 37, Generating the Problem List Feedback: The problem list should have the most active and serious problem first. This new complaint of chest pain is almost certainly a higher priority than his other, more chronic problems.

1. You are running late after your quarterly quality improvement meeting at the hospital and have just gotten paged from the nurses' station because a family member of one of your patients wants to talk with you about that patient's care. You have clinic this afternoon and are double-booked for the first appointment time; three other patients also have arrived and are sitting in the waiting room. Which of the following demeanors is a behavior consistent with skilled interviewing when you walk into the examination room to speak with your first clinic patient? A) Irritability B) Impatience C) Boredom D) Calm

D; Ans: D Chapter: 03 Page and Header: 58, Getting Ready: The Approach to the Interview Feedback: The appearance of calmness and patience, even when time is limited, is the hallmark of a skilled interviewer.

13. A patient is describing a very personal part of her history very quickly and in great detail. How should you react to this? A) Write down as much as you can, as quickly as possible. B) Ask her to repeat key phrases or to pause at regular intervals, so you can get almost every word. C) Tell her that she can go over the notes later to make sure they are accurate. D) Push away from the keyboard or put down your pen and listen.

D; Ans: D Chapter: 03 Page and Header: 58, Getting Ready: The Approach to the Interview Feedback: This is a common event in clinical practice. It is much more important to listen actively with good eye contact at this time than to document the story verbatim. You want to minimize interruption (e.g., answer B). It is usually not appropriate to ask a patient to go over the written notes, but it would be a good idea to repeat the main ideas back to her. You should be certain she has completed her story before doing this. By putting down your pen or pushing away from the keyboard, you let the patient know that her story is the most important thing to you at this moment.

16. A patient complains of knee pain on your arrival in the room. What should your first sentence be after greeting the patient? A) How much pain are you having? B) Have you injured this knee in the past? C) When did this first occur? D) Could you please describe what happened?

D; Ans: D Chapter: 03 Page and Header: 60, Learning About the Patient: The Sequence of the Interview Feedback: When looking into a complaint, it is best to start with an invitation for the patient to tell you in his or her own words. More specific questions should be used later in the interview to fill in any gaps.

19. You are performing a young woman's first pelvic examination. You make sure to tell her verbally what is coming next and what to expect. Then you carry out each maneuver of the examination. You let her know at the outset that if she needs a break or wants to stop, this is possible. You ask several times during the examination, "How are you doing, Brittney?" What are you accomplishing with these techniques? A) Increasing the patient's sense of control B) Increasing the patient's trust in you as a caregiver C) Decreasing her sense of vulnerability D) All of the above

D; Ans: D Chapter: 03 Page and Header: 68, Building a Therapeutic Relationship: The Techniques of Skilled Interviewing Feedback: These techniques minimize the effects of transitions during an examination and empower the patient. Especially during a sensitive examination, it is important to give the patient as much control as possible.

18. A patient tells you about her experience with prolonged therapy for her breast cancer. You comment, "That must have been a very trying time for you." What is this an example of? A) Reassurance B) Empathy C) Summarization D) Validation

D; Ans: D Chapter: 03 Page and Header: 68, Building a Therapeutic Relationship: The Techniques of Skilled Interviewing Feedback: This is an example of validation to legitimize her emotional experience. "Now that you have had your treatment, you should not have any further troubles" is an example of reassurance. "I understand what you went through because I am a cancer survivor myself" is an example of empathy. "So, you have had a lumpectomy and multiple radiation treatments" is an example of summarization as applied to this vignette.

4. A 67-year-old retired janitor comes to the clinic with his wife. She brought him in because she is concerned about his weight loss. He has a history of smoking 3 packs of cigarettes a day for 30 years, for a total of 90 pack-years. He has noticed a daily cough for the past several years, which he states is productive of sputum. He came into the clinic approximately 1 year ago, and at that time his weight was 140 pounds. Today, his weight is 110 pounds. Which one of the following questions would be the most important to ask if you suspect that he has lung cancer? A) Have you tried to force yourself to vomit after eating a meal? B) Do you have heartburn/indigestion and diarrhea? C) Do you have enough food to eat? D) Have you tried to lose weight?

D; Ans: D Chapter: 04 Page and Header: 102, The Health History Feedback: This is important: If the patient hasn't tried to lose weight, then this weight loss is inadvertent and poses concern for a neoplastic process, especially given his smoking history.

12. Mr. Curtiss has a history of obesity, diabetes, osteoarthritis of the knees, HTN, and obstructive sleep apnea. His BMI is 43 and he has been discouraged by his difficulty in losing weight. He is also discouraged that his goal weight is 158 pounds away. What would you tell him? A) "When you get down to your goal weight, you will feel so much better." B) "Some people seem to be able to lose weight and others just can't, no matter how hard they try." C) "We are coming up with new medicines and methods to treat your conditions every day." D) "Even a weight loss of 10% can make a noticeable improvement in the problems you mention."

D; Ans: D Chapter: 04 Page and Header: 104, Health Promotion and Counseling Feedback: Many patients trying to change a habit are overwhelmed by how far they are from their goal. As the proverb says: "A journey of a thousand miles begins with one step." Many patients find it empowering to know that they can achieve a small goal, such as a loss of 1 pound per week. They must be reminded that this process will take time and that slow weight loss is more successful long-term. Research has shown that significant benefits often come with even a 10% weight loss.

18. A 50-year-old body builder is upset by a letter of denial from his life insurance company. He is very lean but has gained 2 pounds over the past 6 months. You personally performed his health assessment and found no problems whatsoever. He says he is classified as "high risk" because of obesity. What should you do next? A) Explain that even small amounts of weight gain can classify you as obese. B) Place him on a high-protein, low-fat diet. C) Advise him to increase his aerobic exercise for calorie burning. D) Measure his waist.

D; Ans: D Chapter: 04 Page and Header: 104, Health Promotion and Counseling Feedback: The patient most likely had a high BMI because of increased muscle mass. In this situation, it is important to measure his waist. It is most likely under 40 inches, which makes obesity unlikely (even to an insurance company). It is important that you personally contact the company and explain your reasoning. Be prepared to back your argument with data. A special diet is unlikely to be of much use, and more aerobic exercise, while probably a good idea for most, is redundant for this individual.

6. You are beginning the examination of a patient. All of the following areas are important to observe as part of the General Survey except: A) Level of consciousness B) Signs of distress C) Dress, grooming, and personal hygiene D) Blood pressure

D; Ans: D Chapter: 04 Page and Header: 109, The General Survey Feedback: Blood pressure is a vital sign, not part of the General Survey.

13. Jenny is one of your favorite patients who usually shares a joke with you and is nattily dressed. Today she is dressed in old jeans, lacks makeup, and avoids eye contact. To what do you attribute these changes? A) She is lacking sleep. B) She is fatigued from work. C) She is running into financial difficulty. D) She is depressed.

D; Ans: D Chapter: 04 Page and Header: 109, The General Survey Feedback: It is important to use all of your skills and memory of an individual patient to guide your thought process. She is not described as sleepy. Work fatigue would most likely not cause avoidance of eye contact. Financial difficulties would not necessarily deplete a nice wardrobe. It is most likely that she is depressed or in another type of difficulty.

11. Susanne is a 27 year old who has had headaches, muscle aches, and fatigue for the last 2 months. You have completed a thorough history, examination, and laboratory workup but have not found a cause. What would your next action be? A) A referral to a neurologist B) A referral to a rheumatologist C) To tell the patient you can't find anything D) To screen for depression

D; Ans: D Chapter: 05 Page and Header: 136, Symptoms and Behavior Feedback: Although you may consider referrals to help with the diagnosis and treatment for this patient, screening is a time-efficient way to recognize depression. This will allow her to be treated more expediently. You may tell the patient you have not found an answer yet, but you must also tell her that you will not stop looking until you have helped her.

14. Adam is a very successful 15-year-old student and athlete. His mother brings him in today because he no longer studies, works out, or sees his friends. This has gone on for a month and a half. When you speak with him alone in the room, he states it "would be better if he were not here." What would you do next? A) Tell him that he has a very promising career in anything he chooses and soon he will feel better. B) Tell him that he needs an antidepressant and it will take about 4 weeks to work. C) Speak with his mother about getting him together more with his friends. D) Assess his suicide risk.

D; Ans: D Chapter: 05 Page and Header: 142, Health Promotion and Counseling Feedback: His lack of interest in usual activities and duration of symptoms should make you suspicious for depression. Despite his very successful academic and athletic performance, you should recognize this last phrase indicating suicide risk. You could ask if he has had thoughts about hurting himself and, if so, how he would carry this out. Ask about firearms and other weapons at home. Adam needs immediate psychiatric referral if these risks are found, or admission to the hospital for observation if referral is not available in a timely fashion.

16. In obtaining a history, you note that a patient uses the word "largely" repeatedly, to the point of being a distraction to your task. Which word best describes this speech pattern? A) Clanging B) Echolalia C) Confabulation D) Perseveration

D; Ans: D Chapter: 05 Page and Header: 145, Techniques of Examination Feedback: Perseveration is the repetition of words or ideas. Echolalia differs in that the patient repeats what is said to him. Clanging is the repetition of the same sounds in different words. Confabulation is making up a story in response to a question. This is sometimes seen in chronic alcohol use with Korsakoff's syndrome.

1. A 19-year-old college student, Todd, is brought to your clinic by his mother. She is concerned that there is something seriously wrong with him. She states for the past 6 months his behavior has become peculiar and he has flunked out of college. Todd denies any recent illness or injuries. His past medical history is remarkable only for a broken foot. His parents are both healthy. He has a paternal uncle who had similar symptoms in college. The patient admits to smoking cigarettes and drinking alcohol. He also admits to marijuana use but none in the last week. He denies using any other substances. He denies any feelings of depression or anxiety. While speaking with Todd and his mother you do a complete physical examination, which is essentially normal. When you question him on how he is feeling, he says that he is very worried that Microsoft has stolen his software for creating a better browser. He tells you he has seen a black van in his neighborhood at night and he is sure that it is full of computer tech workers stealing his work through special gamma waves. You ask him why he believes they are trying to steal his programs. He replies that the technicians have been telepathing their intents directly into his head. He says he hears these conversations at night so he knows this is happening. Todd's mother then tells you, "See, I told you . . . he's crazy. What do I do about it?" While arranging for a psychiatry consult, what psychotic disorder do you think Todd has? A) Schizoaffective disorder B) Psychotic disorder due to a medical illness C) Substance-induced psychotic disorder D) Schizophrenia

D; Ans: D Chapter: 05 Page and Header: 162, Table 5-4 Feedback: Schizophrenia generally occurs in the late teens to early 20s. It often is seen in other family members, as in this case. Symptoms must be present for at least 6 months and must have at least two features of (1) delusions (e.g., Microsoft is after his programs), (2) hallucinations (e.g., technicians sending telepathic signals), (3) disorganized speech, (4) disorganized behavior, and (5) negative symptoms such as a flat affect.

20. Which of the following is commonly seen in aging men? A) Erectile dysfunction in 20% of all men B) Testicles ride higher within the scrotum C) Strong response to visual erotic cues D) Persistent sexual interest

D; Ans: D Chapter: 20 Page and Header: 899, Anatomy and Physiology Feedback: Erectile dysfunction affects about half of elderly men but sexual interest generally remains intact. A decrease in sexual interest may indicate other problems such as depression. Visual cues become less important and tactile stimulation more important. The testicles are positioned lower in the scrotum.

21. Which of the following accompanies decreased ovarian function? A) Increased sleep B) Diminution of sexual interest C) Enlargement of the clitoris D) Decrease in vaginal secretions

D; Ans: D Chapter: 20 Page and Header: 899, Anatomy and Physiology Feedback: Menopause, or the cessation of menses for 1 year, commonly occurs in the late 40s to early 50s. Many experience hot flashes, sweating, chills, anxiety, decreased sleep, and urge incontinence. Dyspareunia is common secondary to decreased vaginal secretions. Sexual interest does not normally decrease. The clitoris and length of the vaginal vault decrease in response to decreased estrogen.

5. An 85-year-old retired teacher comes to your office for evaluation of weakness. You obtain a complete history, perform a thorough physical examination, and order laboratory tests. You diagnose her with hyperthyroidism. Based on her age, which of the atypical symptoms of hyperthyroidism is more likely to be seen? A) Fatigue B) Weight loss C) Tachycardia D) Anorexia

D; Ans: D Chapter: 20 Page and Header: 903, The Health History Feedback: This is an atypical symptom of hyperthyroidism that is more likely to be seen in the older patient.

8. An 89-year-old retired school principal comes for an annual check-up. She would like to know whether or not she should undergo a screening colonoscopy. She has never done this before. Which of the following factors should not be considered when discussing whether she should go for this screening test? A) Life expectancy B) Time interval until benefit from screening accrues C) Patient preference D) Current age of patient

D; Ans: D Chapter: 20 Page and Header: 909, Health Promotion and Counseling Feedback: The current age of the patient is not as important as her actual life expectancy and current health status.

10. You are asked to perform a home safety assessment for an 87-year-old retired farmer who lives by himself. Which of the following is not considered to be an increased risk for falls? A) Loose electrical cords B) Slippery or irregular surfaces C) Chairs at awkward angles D) Bright lighting

D; Ans: D Chapter: 20 Page and Header: 911, Health Promotion and Counseling Feedback: Bright lighting is a recommendation to improve an older person's ability to see all possible things that could result in a fall.

4. A 70-year-old retired auto mechanic comes to your office because his neighbor is concerned about his memory. The patient himself admits to misplacing his keys more often and forgets what he is supposed to buy from the grocery store and where he has parked the car. He denies getting lost in familiar places. Upon further questioning, he states that his wife of 40 years died 8 months ago; his three children live in three different states; and he has limited his activities because the people he interacted with were "his wife's friends, not his." He drinks a six-pack of beer daily; he does not smoke or use illicit drugs. You perform a mini-mental state examination and obtain a total score of 24 out of 28. Based on this information, what is your most likely diagnosis? A) Benign forgetfulness B) Dementia C) Meningitis D) Depression

D; Ans: D Chapter: 20 Page and Header: 911, Health Promotion and Counseling Feedback: The patient has symptoms of depression: His wife died, he has no real social support system, and he has isolated himself from his usual activities. He also drinks a considerable amount of alcohol on a daily basis, which can further depress his mood. Depression can masquerade as dementia in the elderly and must be considered in a patient with memory loss.

26. Claire's daughter brings her in today after Claire fell at her home. Which assessments are indicated at this time? A) Orthostatic vital signs B) Review of her medications C) Assessment of gait and balance D) All of the above

D; Ans: D Chapter: 20 Page and Header: 913, Techniques of Examination Feedback: Falls are common in the elderly and can often result in serious injuries. When assessing the cause of falls, gait and balance should be checked first. Medication, particularly use of more than three, is associated with falls. Vision problems, lower-limb joint problems, and cardiovascular problems such as arrhythmias may be reasonable to search for. Orthostatic vital sign changes should be sought.

1. Which of the following changes are expected in vision as part of the normal aging process? A) Cataracts B) Glaucoma C) Macular degeneration D) Blurring of near vision

D; Ans: D Chapter: 20 Page and Header: 918, Techniques of Examination Feedback: The lens loses its elasticity over time as part of the normal aging process, and the eye is less able to accommodate and focus on near objects; therefore, the patient will be expected to have blurring of near vision.

29. On routine screening you notice that the cup-to-disc ratio of the patient's right eye is 1:2. What ocular condition should you suspect? A) Macular degeneration B) Diabetic retinopathy C) Hypertensive retinopathy D) Glaucoma

D; Ans: D Chapter: 20 Page and Header: 918, Techniques of Examination Feedback: This cup-to-disc ratio means that the cup takes up 50% of the disc, which is abnormally large. This is usually an indication of glaucoma, which is a common cause of visual loss in the elderly. The cup-to-disc changes are not seen in diabetes, hypertension, or macular degeneration. Many elderly do not have regular eye examinations and are not screened for glaucoma.

The nurse is interviewing a male patient who has a hearing impairment. What techniques would be most beneficial in communicating with this patient? a. Determine the communication method he prefers. b. Avoid using facial and hand gestures because most hearing-impaired people find this degrading. c. Request a sign language interpreter before meeting with him to help facilitate the communication. d. Speak loudly and with exaggerated facial movement when talking with him because doing so will help him lip read.

a

A 75-year-old woman is at the office for a preoperative interview. The nurse is aware that the interview may take longer than interviews with younger persons. What is the reason for this? a. An aged person has a longer story to tell. b. An aged person is usually lonely and likes to have someone with whom to talk. c. Aged persons lose much of their mental abilities and require longer time to complete an interview. d. As a person ages, he or she is unable to hear; thus the interviewer usually needs to repeat much of what is said.

a

As the nurse enters a patients room, the nurse finds her crying. The patient states that she has just found out that the lump in her breast is cancer and says, Im so afraid of, um, you know. The nurses most therapeutic response would be to say in a gentle manner: a. Youre afraid you might lose your breast? b. No, Im not sure what you are talking about. c. Ill wait here until you get yourself under control, and then we can talk. d. I can see that you are very upset. Perhaps we should discuss this later.

a

In an interview, the nurse may find it necessary to take notes to aid his or her memory later. Which statement is true regarding note-taking? a. Note-taking may impede the nurses observation of the patients nonverbal behaviors. b. Note-taking allows the patient to continue at his or her own pace as the nurse records what is said. c. Note-taking allows the nurse to shift attention away from the patient, resulting in an increased comfort level. d. Note-taking allows the nurse to break eye contact with the patient, which may increase his or her level of comfort.

a

When taking a history from a newly admitted patient, the nurse notices that he often pauses and expectantly looks at the nurse. What would be the nurses best response to this behavior? a. Be silent, and allow him to continue when he is ready. b. Smile at him and say, Dont worry about all of this. Im sure we can find out why youre having these pains. c. Lean back in the chair and ask, You are looking at me kind of funny; there isnt anything wrong, is there? d. Stand up and say, I can see that this interview is uncomfortable for you. We can continue it another time.

a

A mother brings her 28-month-old daughter into the clinic for a well-child visit. At the beginning of the visit, the nurse focuses attention away from the toddler, but as the interview progresses, the toddler begins to warm up and is smiling shyly at the nurse. The nurse will be most successful in interacting with the toddler if which is done next? a. Tickle the toddler, and get her to laugh. b. Stoop down to her level, and ask her about the toy she is holding. c. Continue to ignore her until it is time for the physical examination. d. Ask the mother to leave during the examination of the toddler, because toddlers often fuss less if their parent is not in view.

b

Receiving is a part of the communication process. Which receiver is most likely to misinterpret a message sent by a health care professional? a. Well-adjusted adolescent who came in for a sports physical b. Recovering alcoholic who came in for a basic physical examination c. Man whose wife has just been diagnosed with lung cancer d. Man with a hearing impairment who uses sign language to communicate and who has an interpreter with him

c

A pregnant woman states, I just know labor will be so painful that I wont be able to stand it. I know it sounds awful, but I really dread going into labor. The nurse responds by stating, Oh, dont worry about labor so much. I have been through it, and although it is painful, many good medications are available to decrease the pain. Which statement is true regarding this response? The nurses reply was a: a. Therapeutic response. By sharing something personal, the nurse gives hope to this woman. b. Nontherapeutic response. By providing false reassurance, the nurse actually cut off further discussion of the womans fears. c. Therapeutic response. By providing information about the medications available, the nurse is giving information to the woman. d. Nontherapeutic response. The nurse is essentially giving the message to the woman that labor cannot be tolerated without medication.

b

A woman is discussing the problems she is having with her 2-year-old son. She says, He wont go to sleep at night, and during the day he has several fits. I get so upset when that happens. The nurses best verbal response would be: a. Go on, Im listening. b. Fits? Tell me what you mean by this. c. Yes, it can be upsetting when a child has a fit. d. Dont be upset when he has a fit; every 2 year old has fits.

b

During a visit to the clinic, a patient states, The doctor just told me he thought I ought to stop smoking. He doesnt understand how hard Ive tried. I just dont know the best way to do it. What should I do? The nurses most appropriate response in this case would be: a. Id quit if I were you. The doctor really knows what he is talking about. b. Would you like some information about the different ways a person can quit smoking? c. Stopping your dependence on cigarettes can be very difficult. I understand how you feel. d. Why are you confused? Didnt the doctor give you the information about the smoking cessation program we offer?

b

The nurse has used interpretation regarding a patients statement or actions. After using this technique, it would be best for the nurse to: a. Apologize, because using interpretation can be demeaning for the patient. b. Allow time for the patient to confirm or correct the inference. c. Continue with the interview as though nothing has happened. d. Immediately restate the nurses conclusion on the basis of the patients nonverbal response

b

The nurse is conducting an interview with a woman who has recently learned that she is pregnant and who has come to the clinic today to begin prenatal care. The woman states that she and her husband are excited about the pregnancy but have a few questions. She looks nervously at her hands during the interview and sighs loudly. Considering the concept of communication, which statement does the nurse know to be most accurate? The woman is: a. Excited about her pregnancy but nervous about the labor. b. Exhibiting verbal and nonverbal behaviors that do not match. c. Excited about her pregnancy, but her husband is not and this is upsetting to her. d. Not excited about her pregnancy but believes the nurse will negatively respond to her if she states this.

b

A 17-year-old single mother is describing how difficult it is to raise a 3-year-old child by herself. During the course of the interview she states, I cant believe my boyfriend left me to do this by myself! What a terrible thing to do to me! Which of these responses by the nurse uses empathy? a. You feel alone. b. You cant believe he left you alone? c. It must be so hard to face this all alone. d. I would be angry, too; raising a child alone is no picnic.

c

A nurse is taking complete health histories on all of the patients attending a wellness workshop. On the history form, one of the written questions asks, You dont smoke, drink, or take drugs, do you? This question is an example of: a. Talking too much. b. Using confrontation. c. Using biased or leading questions. d. Using blunt language to deal with distasteful topics.

c

During an interview, a woman says, I have decided that I can no longer allow my children to live with their fathers violence, but I just cant seem to leave him. Using interpretation, the nurses best response would be: a. You are going to leave him? b. If you are afraid for your children, then why cant you leave? c. It sounds as if you might be afraid of how your husband will respond. d. It sounds as though you have made your decision. I think it is a good one.

c

A man has been admitted to the observation unit for observation after being treated for a large cut on his forehead. As the nurse works through the interview, one of the standard questions has to do with alcohol, tobacco, and drug use. When the nurse asks him about tobacco use, he states, I quit smoking after my wife died 7 years ago. However, the nurse notices an open pack of cigarettes in his shirt pocket. Using confrontation, the nurse could say: a. Mr. K., I know that you are lying. b. Mr. K., come on, tell me how much you smoke. c. Mr. K., I didnt realize your wife had died. It must be difficult for you at this time. Please tell me more about that. d. Mr. K., you have said that you dont smoke, but I see that you have an open pack of cigarettes in your pocket.

d

A patient has finished giving the nurse information about the reason he is seeking care. When reviewing the data, the nurse finds that some information about past hospitalizations is missing. At this point, which statement by the nurse would be most appropriate to gather these data? a. Mr. Y., at your age, surely you have been hospitalized before! b. Mr. Y., I just need permission to get your medical records from County Medical. c. Mr. Y., you mentioned that you have been hospitalized on several occasions. Would you tell me more about that? d. Mr. Y., I just need to get some additional information about your past hospitalizations. When was the last time you were admitted for chest pain?

d

A woman has just entered the emergency department after being battered by her husband. The nurse needs to get some information from her to begin treatment. What is the best choice for an opening phase of the interview with this patient? a. Hello, Nancy, my name is Mrs. C. b. Hello, Mrs. H., my name is Mrs. C. It sure is cold today! c. Mrs. H., my name is Mrs. C. How are you? d. Mrs. H., my name is Mrs. C. Ill need to ask you a few questions about what happened.

d

During an examination of a 3-year-old child, the nurse will need to take her blood pressure. What might the nurse do to try to gain the childs full cooperation? a. Tell the child that the blood pressure cuff is going to give her arm a big hug. b. Tell the child that the blood pressure cuff is asleep and cannot wake up. c. Give the blood pressure cuff a name and refer to it by this name during the assessment. d. Tell the child that by using the blood pressure cuff, we can see how strong her muscles are

d

During an interview, a parent of a hospitalized child is sitting in an open position. As the interviewer begins to discuss his sons treatment, however, he suddenly crosses his arms against his chest and crosses his legs. This changed posture would suggest that the parent is: a. Simply changing positions. b. More comfortable in this position. c. Tired and needs a break from the interview. d. Uncomfortable talking about his sons treatment

d

During an interview, the nurse states, You mentioned having shortness of breath. Tell me more about that. Which verbal skill is used with this statement? a. Reflection b. Facilitation c. Direct question d. Open-ended question

d

In using verbal responses to assist the patients narrative, some responses focus on the patients frame of reference and some focus on the health care providers perspective. An example of a verbal response that focuses on the health care providers perspective would be: a. Empathy. b. Reflection. c. Facilitation. d. Confrontation.

d

The nurse asks, I would like to ask you some questions about your health and your usual daily activities so that we can better plan your stay here. This question is found at the __________ phase of the interview process. a. Summary b. Closing c. Body d. Opening or introduction

d

Which statement best describes a proficient nurse? A proficient nurse is one who: a. Has little experience with a specified population and uses rules to guide performance. b. Has an intuitive grasp of a clinical situation and quickly identifies the accurate solution. c. Sees actions in the context of daily plans for patients. d. Understands a patient situation as a whole rather than a list of tasks and recognizes the long-term goals for the patient.

d


Ensembles d'études connexes

Chapter 7 - Criminal Law and Cybercrime

View Set

Macro Exam Worksheet - Exchange rates

View Set

ИКТ экзамен 1 часть

View Set

General Biology 2 - MIDTERMS (Final Test)

View Set